Sunteți pe pagina 1din 142
‘SAT 2006 May Answers and Explanations 2006 May SAT AFT (Online Course Test1) Section1: Essay Essay Prompt ‘Think carefully about the issue presented in the fllowing excerpt and the assignment below It is not that people dislike being part of @ community; itis just that they care about the individual freedoms more. People value neighbortiness and sacia interaction—unti being part of a group requires them to limit thelr freedom for the larger good of the group. But a community or group cannot function cfectively unless people are willing to Set aside thelr personal interests [Adapted from Warren Johnson, The Future Is Not What Ik Usee Te Be ‘Assignment: oes the success of a community—whether itis a class, @ team, a family, 3 nation, or any other ‘9roun—depend upon people's willingness to iit their personal interests? Plan and write an essay in which you develop your point of view on this issue, Support your position with reasoning and exerples taken fram your reading, studies, experience, ot observations Sample Essay - Score of 6 | belicve that the idea of a community, as Warren Johnson stated in hisThe Future Js Not What It Used To Be, does depend upon its members’ willingness to “set aside their personal interests.” This often requires a strong common goal, bond, experience, or circumstance to tie the humans together in this group, in a place where they value the community enough that they are each willing to sacrifice part of themselves for it, Asa highschooler, I see this on a small scale every day. My school has roughly 300 students, and the idea of “community” is impressed upon us as being a core tenent of the school’s, values. To cement this the faculty try o give us common experiences, ideas and beliefs to hhold us together. They recogniar each student’s diversity and complexity — we arehurman after all — and the fact that it would be impossible for each of us to find a place with our vast range of knowledge, personalities, and beliefs without a common need to hold FromSAT Online Course CUUS WABI ‘SAT 2006 May Answers and Explanations together. Two ways these commonalities are given to us is by our requirement to attend Chapel everyday and show up at 8 am, regardless if we have no class that period, By altending Chapel we come to find common ground, even if our beliefs are different, by showing up at 8 we all can gripe together of our exhaustion, However, this community can and does fall apart because for any community to perfectly function requires each member fo completely want the community, want their functioning role within it, In a forced community such as 2 highschool, there will always be those displaced from the community because many simply are not willing to entirely sacrifice or limit themselves for a mainstream of a community. This is also clearin pop culture ~ the songs are limited, the clothes are limited, and each magazine cover in the grocery store has the shining face of Jessica Simpson or Angelina Jolie on it. ‘Then there are mainstream rejects, by choice or otherwise, They form their own communities, but with their commonalities bonding them — from mohawks to politics to emotional states. Another example of communities by limiting ideas is communism, Asthe world has seen, achieving communism to its best potential is nearto impossible when applied on a large scale, Humans are too complex to expect each one to entirely cooperate to such a particular brand of ideas, especially in a situation when this cooperation is essential for the society to even remotely work. It requires the desire of each ‘Score Explanation Displaying clear and consistent mastery, this essay effectively and insightfully develops « point of view, explaining that each community “does depend upon its members’ willingness to ‘set aside their personal interests,” but acknowledging that “many simply arenot willing to entirely sacrifice or limit themselves for a mainstream of a community.” ‘The essay demonstrates outstanding critical thinking a: it uses the clearly appropriate example of “a forced community”—the writer’s high school—and other evidence to support the position. Well organized and clearly focused, the essay shows clear coherence and smooth progression of ideas as it explores the complexities of the topic (Mf soho! ‘has roughh 30 students, ana the idea 0; ‘community’ is impressea upon us as being a core teneni 0) the schoois values. Tc cemem this the faculty try to give us common experiences, ideas ana belief to hola us together... However, this community can ana does fads apart), ‘oe: essay exhibits a varied, accurate, and apt vocabulary and demonstrates meaningful variety in sentence structure (/fumans are too complex to expec each one to entirely cooperate to such a particular brana 0) ideas, especially ina situation wher this cooperation is essentias for the society ts even remotely work). This, outstanding essay receives 4 score of 6. Sample Hatay -Score of 5 Community is defined as a group of people who work together and care about eachothers wellbeing. Fora community to funetion properly it must make sacrifices in individual lives to ensure the development and success of the entire idea. Sacrifices are necessary because if each person is not looking out for others, not caring about the whole effect, and not thinking their contribution counts than the whole community cannot work, FromSAT Online Course Sith CUUS WABI ‘SAT 2006 May Answers and Explanations In the neighborhood where I grew up there was a community organization called “Neighborhood Crime Watch.” This group of people would patrol our streets at night to Keep us all safe. Many people chose not to participate because it took quite some time out of their nightly cyele of sleep. One night there was a group of young boys, who had been ata party, and they decided to break into our neighbors house. Due to the Crime Watch program they were stopped before any harm was done. When people in the community ‘watch out for one another the effects are well appreciated and noticed. ‘The whole idea of community is to link together @ group of people who have the same basic needs for unity. There are some people who think they are too good to be counted in with their neighbors in the community. Most times, the people who have that mindset don’t see the picture of the overall effect. For quite some time in my neighborhood, there wasan old man who didn’t want to be part of “Crime Watch.” He thought it was pointless and he decided he locked his doors, and that was good enough, So one night, his house got robbed and some precious valuables were stoled. After that experience he decided that crime watch actually had 2 real purpose and maybe if be had been patroling, he could have stopped what happened to his house. When he sawthe overall effect and decided to put aside just a few hours of his time, many people joined the program because he was sucha big influence, Another instance of erime watch was when a single lady moved into the neighborhood. ‘She was quite slender and didn’t think she was strong enough to stop anybody who could inflict harm and distress on others. She grudgingly accepted to do herduty, not thinking it ‘would matter. She may have not been strong, but she was smart enough to call the cops and some bigger guy neighbors when she saw guy getting beat up. She saved a mans lite Community is an important aspect of all our lives. ‘Score Explanation This essay demonstrates strong critical thinking by effectively developing a point of view (For a community to finction properly it must make sacrifees in individual lives to ensure the development and success ofthe entire ided) and using an appropriate extended example as support. Well organized and focused on the example of the "Neighborhood Crime Watch,” the essay displays coherence and progression of ideas (4/fer shat experience he decided that crime watch actually hada real purpose and maybe if he had been patroling, he could have stopped what happened to his house When he saw the overall effect and decided to put aside just a few hours of his time, many people joined the program because he was such a big influence), Despite some minor errors in grammar, usage, and mechanics, the response exhibits facily in the use of language and variety in sentence structure (She was quite slender and didn't think she was strong enough to stop anybody who could inflit harm and distress on others. To earn score of 6, the essay needs to demonstrate greater progression of ideas across the paragraphs: each body paragraph makes more or less the same point. Demonstrating reasonably consistent mastery, this essay earnsa score of 5 FromSAT Online Course Sk SWAB ‘SAT 2006 May Answers and Explanations Sample Eatay -Scare of 5 Our world is full of small communities. Almost every activity one can get involved in becomes a community in one way or another, whether it be a school, team, church, or club, The idea of Warren Johnson that “... A community or group cannot fimetion effectively unless people are willing to set aside their personal interests,” is completely innacurate First of all, a community can still be successful even if its members’ freedoms are not limited. Pethaps the greatest example of all time is our nation. The United States of America was founded with the protection of its citizens’ individual freedoms as its umber one objective. The founding colonists didn’t set theit personal freedoms aside, but made a list of them. The Bill of Rights is simply all the personal freedoms colonists, ‘wished fo uphold cnumerated, This unprecedented protection of inalienable rights did not prevent our country from becoming successful. The Bill of Rights helped our country prosper into the strongest and most successful nation in the world. So, it’s not necessary that individuals limit their freedoms in order for their community to prosper. In addition, limiting the rights of a communities members may actually harm the communities success, Many people work solely for selfish reasons and may only find the ‘motivation to work hard if he or she will benefit from the outcome. How many times have you heard the question, “Whats in it for me?” Limiting peoples freedoms would take away a motivation to work. Let's face it, Money is a motivator. People work for the personal interest of financial gain. If money was taken away along with other selfish interests, communities would cease to be successful Allin all, the idea that personal interests should be set aside in order for ¢ community to be successful is false. Our nation, the most successful country in the world, is one of the biggest examples that that claim is bogus. Personal interests are the motivation behind success. Take it away and communities’ success would be harmed, not aided. ‘Score Explanation Demonstrating strong critical thinking, this essay develops a point of view (Ail in all, the ‘idea that personal interests should be set aside in order for a community to be successfid és fase) and provides appropriate reasons and examples from history and personal observations to support this position. The essay is well organized and focused, demonstrating coherence and progression of ideas (The United States of America was Jfaunded with the protection of its citizens’ individual freedoms as its mumber one objective. The founding colonists didn't set their personal freedoms aside, but made a list ofthem). Using appropriate vocabulary, the essay exhibits facility in the use of language, and variety in sentence structure appears throughout (Many people work solely far selfish reasons and may only find the motivation to work hard if he or she will benefit from the ‘outcome. In order to receive a score of 6, the essay needs to use critical thinking and additional evidence to develop the examples—particularly the “financial gain” exampl—more fully. Demonstrating reasonably consistent mastery, this response receives a score of 5, FromSAT Online Course Sk SWAB ‘SAT 2006 May Answers and Explanations Section3: Critical Reading Online - Practice Test #1 Her political ~—---- came from her Vision of the nation, a vision that Included and drew strenath from every social constituency. () autonomy . . sweeping (9) reticence... mystical (©) triumph . . restricted (©) rebetion . conventional WIE) success. . comprehensive ANSWERS AND EXPLANATIONS Explanation for Correct Answer E : Choice (E) Is correct. In this context, “success” means the attainment of favor or prominence. “Comprehensive” means covering complately or broadly. The structure of the sentence suggests that the text after the comma further explains the type of vision that is mentioned in the first half of the sentence, Therefore, the word that fits in the second blank should describe a vision that “included and drew strength from every social constituency.” The term "comprehensive" makes sense because a vision that Includes every constituency, or group of people, Is certainly complete and broad. Further, itis likely that this type of all-inclusive vision would resuk in a politcian’s Explanation for Incorrect Answer A: CChoice (A) is incorrect. "Autonomy" Is freedom or Independence. “Sweeping” means moving oF extending over a wide area. The structure of the sentence suggests that the text after the comma further explains the type of vision that is mentioned in the first half ofthe sentence. Therefore, the word that fits in the second blank should describe a vislon that “included and drew strength from every social constituency." While a vision that Includes every constituency, or group of people, could be described as sweeping, this type of vision would not necessarily lead to e politician's autonomy, or Indepencence. On the contrary, a politician who has @ sweeping vision would most likely work with various citizens +z er than work independently Explanation for Incorrect Answer B Choice (B) is incorrect, To be reticent Is to be silent or restrained in expression, present sentence suggests th ni, oF appearance. "My he text after the comma further explains I" means mysterious. The structure of the type of visio FromSAT Online Course Sih CUUS HA ‘SAT 2006 May Answers and Explanations is mentioned in the first half of the sentence. Therefore, the word that fits in the second blank should describe a vision that “included and drew strength from every social constituency.” Although a politician could conceivably be reticent, or restrained, in an effort to keep his or her vision a mystery, there Is no logical connection between @ Vision that is mystical and one that inckides every constituency, or group of people. Explanation for Incorrect Answer Cz Cchoice (C) is incorrect, “Triumph” means vietory. "Restricted" means limited. The structure of the sentence suggests that the text after the comma further explains the type of vision that is mentioned in the first half of the sentence. Therefore, the word that fits in the second blank should describe a vision that "Included and drew strené from every social constituency.” Itis illogical to suggest that a vision that is restricted Includes every constituency, oF group of people, Additionally, 2 restricted vision might rot leat politician's triumph, as many citizens might reject or dislike such a vision. Explanation for Incorrect Answer D: Choice (D) is incorrect. "Rebellion" is eppesition to one in authority or dominance. “Conventional” means based on or according with an established practice or technique. ‘The structure of the sentence suggests that the text afte .¢ comma further explains the type of vision that Is mentioned in the frst half of the sentence, Therefore, the word fits in the second blank should describe a vision that “inclided and drew strength from every social constituency.” It's possible that a vision that Includes every constituency, oF group of people, could be seen as conventional, but there is ne reason to believe that such a vision would be the source of a politician's rebellion [As one would expect, the reclusive poet of privacy. ~ public appearances and invasions (8) rewarded © endorsed © neglectes ©) invites MAE) detested ANSWERS AND EXPLANATIONS: Explanation for Correct Answer E Choice (E) Is correct. To detest something is to hate or loathe it. The sentence Gescribes the poet as reclusive, or soltary and withdrawn from society. It makes sense to “expect” that a poet who is withdrawn from society would hate er loathe being out in public and having his or her privacy invaded, so the term “detested” logically completes From SAT Online Co arse i CLUS HAL ‘SATT2006 May Answers and Explanations the sentence Explanation for Incorrect Answer A : Choice (A) Is Incorrect, & reward is something oven for a special achievement or in exchange for a service. The sentence describes the post as reclusive, or solitary and withdrawn from society. Its iogieal to suggest that one could givea reward to a public appearance or an Invasion of privacy. Further, there is no reason to “expect” that 2 post who is withdrawn from society would want to reward such appearances or Invasions. Rather, the poet Ikely “detested,” or hated, them. Explanation for Incorrect Answer Choice (B) is incorrect. “Endorsed” means approved or supported. The sentence describes the poet as reclusive, or soltary and withdrawn from society. A person who prefers to be alone and to keep his or her lfe private would not be expected to endorse, oF approve of, public appearances and invasions of privacy. On i 1e contrary, # is type of person would likely detest, or hate, such things. Explanation for Incorrect Answer C1 Choice (C) is incorrect. To neglect means te give ite attention or respect to something The sentence describes the poet as reclusive, or soltary and withdrawn from soclety. While it is true that @ person who prefers to be alone and to keep his orher life private ‘might neglect, or pay litle attention to, public appearances end invasions of privacy, it 's most logical to “expect” that this type of person would completely avoid such things—not just pay ite attention to them. Explanation for Incorrect Answer D : Choice (D) is Incorrect, In this context, to Invite means to welcome or accept with pleasure the occurrence or presence of something. The sentence describes the poet as reclusive, or solitary and withdrawn from society. A person who prefers to be alone and to keep his or herlife private would probably net invite, or welcome, public appearances and invasions of privacy. On the contrary, this type of person would likely etest, or hate, such things, and would make every effort to completely avoid them. ‘Tumn-of-the-century actress Sarah Bernhardt had so ------~ a talent that she ~ audiences with her diverse and utterly convincing characteriations. (A) unrealized . . enchanted 18) protean .. bedazzled () eclectic... wearies FromSAT Online Course Sih CUUS HA ‘SAT 2006 May Answers and Explanations ©) quixotic. . confoundes (© mediocre... spellbound ANSWERS AND EXPLANATIONS: Explanation for Correct Answer! Choice (B) is correct. *Protean” means displaying great variety or diversity, To bedazse ‘means to impress or enchant. The structure of the sentence implies a cause and effect relationship: because Sarah Bernhardt had a certain kind of talent, she had a certain effect on her audiences. Therefore, the missing words need to illustrate this type of relationship. Audiences would certainly be be¢azzled, or enchanted, with someone who has @ protean, or diverse, talent, Further, the phrase “her diverse and utterly convincing characterimtions” reinforces the Idea that her talent was protean. Explanation for Incorrect Answer At Choice (A) is incorrect. “Unrealized” means not made actual or real. To enchant means to attract, delight, or move deeply. The structure of the sentence implies @ cause anc effect relationship: because Sarah Bernhardt had a certain kind of talent, she had a certain effect on her audiences. Therefore, the missing words need to illustrate this type of relationship. It is unlikely that audiences would be enchanted, or delignted, by someone whose talent was unrealized, or not Unrealized talent could accomplish "utterly convincing characteristions.” al. tis also unlikely that someone with Explanation for Incorrect Answer C: Choice (C) is incorrect. “Eclectic” means composed of elements or items drawn from various sources. “Wearled" means tired. The structure of the sentence Implles a cause and effect relationship: because Sarah Bernhardt had a certain kind of talent, she had 2 certain effect on her audiences. Therefore, the missing Words need to illustrate this type of relationship. While the phrase “her diverse and utterly convincing characteriations” might suggest that Sarah Sernhardt’s talent was eclectic, or drawn, from various sources, it éoes not make sense to say that audiences would be wearled, or tired, as a result of watching Bernhardt’s performances. Explanation for Incorrect Answer D : Choice (D) Is Incorrect, “Quixotic” means foollshly impractical or unpredictable. “confounded” means confused. The structure af the sentence implies a cause and effect tain king of talent, she had a certain effect on her audiences, Therefore, the missing words need to illustrate this type of relationship. Although itis logical to suggest that audiences might have been confused by someone whe was quixotic, it is somewhat illogical to describe one’s talent as foolishly impractical. Furthermore, there is no reason to believe that audiences would have been confounded specifically by Bernhardt’s “utterly convincing characteriattions. relationship: because Sarah Bernhardt had a c FromSAT Online Course CUUS WABI ‘SAT 2006 May Answers and Explanations Explanation for Incorrect Answer E+ Choice (F) is incorrect. "Mediocre” means having moderate or low quality, value, ability, oF performance. “Spellbound” means held by @ strong compelling influence or action. The structure of the sentence implies a cause and effect relationship. because Sarah Bernhardt had a certain kind of talent, she had a certain effect on her audiences, Therefore, the missing words need to illustrate this type of relationship, Its. Lnikely that audiences would be spellbound, or held by some strong attraction, by someone who has a mediocre talent, It's also unlikely that someone with a low abilty could accomplish “utterly convincing characteriztions. In the classroom, Carol was unusually ~; on the playground, however, she became as intractable as the other children. (8) optimistic (©) mercurial (©) magnanimous (©) taciturn © docile ANSWERS AND EXPLANATIONS Explanation for Correct Anwar E : Choice (F) is correct. ‘Docile” means easily taught or manageé. The wore "however" and the structure of the sentence suggest that Carol behaved the same way as the other children when she was on the playground ("as intractable as the other children”) but behaved in a very different way than the ther children when she was in the classroom, Therefore, the missing word should describe a type of behavior that clrectly contrasts with being intractable, or difficult to manage. The term “docile” logically completes the sentence: Carol was easy to manage in the classroom, but on the playground she became as dificult to manage as the other children, Explanation for Incorrect Answer A Choice (A) is incorrect. To be “optimistic” means to be inclined to anticipate the best possible outcome of something, The word “however” and the structure of the sentence suggest that Carol behaved the same way as the other children when she was on the playground ("as intractable as the other children”) but behaved in 8 very different way than the other chileren when she was in the classroom. Therefare, the missing wore should describe a type of behavior that directly contrasts with being intractable, or dificult to manage. The term "optimistic” is not the best choice; there is no contrast between being optimistic, or hoping for the best, end being dificult to manage. Rather, 2 child could be optimistic and intractable at the same time. FromSAT Online Course Sih CUUS HA ‘SAT 2006 May Answers and Explanations Explanation for Incorrect Answer Choice (B) is incorrect. "Mercurial" means to have quickly changing moo¢s. The word however" and the structure of the sentence suggest that Carol behaved the same way {as the other children when she was on the playground ("as intractable as the other, children") but behaved in a very different way than the other children when she was in the classroom, Therefore, the missing word should describe a type of behavier that directly contrasts with being intractable, or difficult to manage. The term “mercurial” does not necessarily contrast with the term “intractable”; Ifa child's moods change Quickly in the classrcom, that child might be described as difficult to manage. Explanation for Incorrect Answer C Choice (C) is incorrect. "Magnanimous" means showing or suggesting allofty or ‘courageous spirit. The word "however” and the structure of the sentence suggest that Carol behaved the same way as the other children when she was on the playground (Cas intractable as the other children”) but behaved in a very different way than the other children when she was in the classroom. Therefore, the missing word should describe a type of behavior that directly contrasts with belng intractable, or dificult to manage. The term "magnanimous" is not the best choice; there is no logical connect between being lofty er courageous and being intractable, or dificult to manage, so there Is no direct contrast: Explanation for Incorrect Answer Dt Choice (D) is incorrect. “Taciturn” means temperamentally silent or disinclined to talk ‘The word *however* and the structure of the sentence suggest that Carol behaved the same way as the ather children when she was on the playground ("as intractable as the other children”) but behaved in a very different way than the other children when she was in the classroom. Therefore, the missing word should describe a type of behavior that directly contrasts with being intractable, oF dificult to manage. The term “tacturn” oes not necessarily contrast with the term “intractable”; if a child is unusually disinclined to talk or temperamentally silent, that child might be described as dificut to manage. Bubble gum is not a topic usually treated seriously, so itis appropriate that this new book tracing the cultural history of bubble gum has a —---- tone. (4) morbid © catnartic (©) pedantic Y flippant © ‘reticent FromSAT Online Course S&{h CU ‘SAT 2006 May Answers and Explanations ANSWERS AND EXPLANATIONS: Explanation for Correct Answer D : Choice (D) is correct. “Rinpant” means lacking respect or seriousness. The tone of a book is its style or manner of expression. The sentence states thet a new book that iscusses the cultural history of bubble gum has a tone that is appropriate, or suitable, for a topic that Is not usually “treated seriously." Therefore, since the topic of bubble gum is usually not taken seriously, it would certainly be appropriate for the book to have a tone that is flippant, or lacking seriousness. Explanation for Incorrect Answer A: Choice (A) is incorrect, "Morbid means gruesome or gloomy. The tone of a book is Its style or manner of expression. The sentence states that a new book ths cultural history of bubble gum has a tone that is appropriate, or suttable, fora topic that is not usually "treated seriously"—there is no reason to believe that a morbid, or jscusses the gloomy, tone would be appropriate for such a topic. On the contrary, a humorous or lightheartes tone would be appropriate for atopic that is not usually treated seriously Explanation for Incorrect Answer B Choice (B) is incorrect. "Cathartic” means producing a spiritual renewal or release from tension. The tone of @ book is its style or manner of expression. The sentence states that a new book that discusses the cultural history of bubble gum has a tone that is appropriate, or sultable, for tople that Is not usually “treated serlously "It does not make sense to describe a book about bubble gum as having a “cathartic tone.” Instead, humorous or lighthearted tone would be appropriate for the book Explanation for Incorrect Answer C+ Choice (C) is incorrect. "Pedantic” means unimaginative and self-righteous. The tone of a book is its style or manner of expression. The sentence states that a new book that Giscusses the cultural history of bubble gum has a tone that is appropriate, or sultable, for a topic that Is not usually “treated seriously." An unimaginative, self-righteous tone would clearly be Inappropriate for a tople such as this one, Rather, a humorous or lighthearted tone would be appropriate for such a topic. Explanation for Incorrect Answer E Choice (E) is incorrect. *Reticent” means silent ar reserved. The tone of a book is its style or manner of expression, The sentence states that @ new book that iscusses the ccutural history of bubble gum has a tone that is appropriate, or suitable, for a topic that is not usually “treated seriously.” It does not make sense to describe the tone of a book as reticent, or silent. Moreover, @ humorous or lighthearted tone would be FromSAT Online Course CUUS WABI ‘SAT 2006 May Answers and Explanations appropriate for the book Jamake Highwater manages to touch on the arts of almost every American Indian nation in one reasonably sized book that makes up for its occasional lack of with its remarkable (8) specificity . . detail (©) discontinuity .. concreteness (©) jpttiness .. inaccessbiity V1) profuncity .. inclusiveness (© uniqueness .. comparability [ANSWERS AND EXPLANATIONS, Explanation for Correct Answer D : Choice (D) is correct, "Profundlty” means having intellectual depth anc insight, To be Inclusive isto be broad in scope, covering all relevant material. The phrase “manages ich on” suggests thet Kighwater briefly mentions or discusses “the arts of almost every American Incian nation," but does not necessarily go inte much detail. Therefore, It is logical to say that Highwater’s book has an occasional lack of profundity, or Intellectual depth, but "makes up,” or compensates, for this éeficiency with its wo remarkable inclusiveness, or scope. Explanation for Incorrect Answer A: Choice (A) Is incorrect, “Specificity” means being specie, or explicit end definite Detail" is extended treatment of ar attention to particular items. The phrase "manages to touch on” suggests that Highwater briefly mentions or discusses “the arts of almost every American Incian nation,” but does not necessarily go into much detail. Because Highwater merely touches on the arts of almost every American Indian nation, it does not make sense to say that the book "makes up,” or compensates, for a lack of specificity with its remarkable detail, or attention to particular tems. Explanation for Incorrect Answer Choice (B) Is incorrect, “Discontinuity” is a lack of continuity or coherence, To be concrete is to be characterized as specific or particular. The phrase "manages to touch fon" suggests that Highwater briefly mentions or discusses “the arts of almost every ‘American Indian nation,” but does not necessarily go into much deta, If the book lacks discontinuity, it has continulty—this Is typically regarded as @ good thing, soit does not make sense to suggest that the book needs to "[make] up for” its continuity, Additionally, iis ilogical to suggest that the book has remarkable "concreteness," or specificity, if Highwater only briefly touches on certain topics, FromSAT Online Course Sih CUUS AB ‘SAT 2006 May Answers and Explanations Explanation for Incorrect Answer C+ Choice (C) is incorrect. "Loftness” means having @ haughty, overbearing manner. In this context, inaccessible means not capable of being understood or appreciated Lacking loftiness would likely be considered a good thing—the book would not have @ haughty and overbearing tone—so it does not make sense to suggest that Highwater Would need to "[make] up for” such a lack, Additionally, if the book displays “remarkable inaccessibility,” itis extraordinarily difficult to understand; itis illogical to suggest that this inaccessibilty Is desirable or that It would make up for anything the book lacks. Explanation for Incorrect Answer E Choice (E) Is incorrect. In this context, to be unique means te be original or one of a kind. "Comparability" is @ quality of being comparable, or similar, to something. A book that has an occasional lack of uniqueness, or originality, is not one of a kind—it could be compared to other books that cover similar topies or are writen In a similartone or format. A book that "makes up,” or compensates, for its occasional lack of uniqueness Would likely have qualities or features that make it different or that make it stand out from other books. Therefore, having remarkable comparability, or being similar to something else, would not make up fora book's lack of originally. ‘Though Judd is typically ------- and reserved in social gatherings, at last night's reception he spoke and acted with uncharacteristic ©) loquacious . .aleerity {®) querulous . . languor (©) disaffected «resentment (©) diplomatic. . decorum WI) ditfident .. aplomb ANSWERS AND EXPLANATIONS Explanation for Correct Answer E Choice (E) Is correct. *Diffident" means shy or reserved. "Aplomb” is confidence or self. ‘assurance, The sentence refers to @ specific time when Judd’s behavior was different from his typical, oF usual, behavior. If Judd is usually reserved in social gatherings itis reasonable to suggest that he is elso shy in such situations, so the term “difident™ makes sense in the first blank. At the reception, Judd’s behavior was Uncharacteristic—that Is, it differed from his usual behavior, It would certainly be uncharacteristic for Judd to speak and act with confidence In @ social gathering if he Is Usually shy and reserved in such situations, so the term “aplomb logically completes the sentence. FromSAT Online Course kh CUUS HERS, ‘SAT 2006 May Answers and Explanations Explanation for Incorrect Answer A: Choice (A) is incorrect. "Loquacious” means talkative. “Alacrity” is liveliness or eagerness. The sentence refers to a specifi time when Judd’s behavior was different from his typical, or usual, behavior, which includes being reserved in social situations. If Judd is typically reserved in such situations, it does not make sense to suggest that he is also loquacious, or talkative. Additionally, if Judd was usually talkative in social gatherings, speaking with alacrity, or eagerness, at the reception would not be unusual behavior, Explanation for Incorrect Answer B Choice (B) is incorrect. "Querulous” means habitually complaining. *Languor” is weariness or weakness of the bedy or mind. The sentence refers to a specific time when Judd’s behavior was different from his typical, or usual, behavior, which includes being reserved in social situations. If Jude is typically reserved—that is, if he is usually restrained In speaking—It |s somewhat illogical to suggest that he also often complains in social situations. Furtnermore, someone who fs usually reserved might speak in a way that could be described as weak or weary, sa it would not necessarily be Uuncharacteristc for Judd to speak with languor at the reception. Explanation for Incorrect Answer C Choice (C) is Incorrect. "Disaffected” means discontent or resentful, “Resentment” is a feeling of displeasure or ill wil toward someone or something, The sentence refers to a specific ime when ludd’s behavior was different fram his typical, or usual, behavior, len includes being reserved in social situations, The term “disaffected” might make sense in the first blank, because Its possible that someone who Is reserved in social gatherings could be resentful, but the term "resentment" does not logically complete the sentence. If Juda is usually reserved and resentful, speaking with resentment, or ispleasure toward something, would not be uncharacteristic, Explanation for Incorrect Answer D: Choice (D) is Incorrect, “Diplomatic” means being polsed or tactful, “Decorum is propriety and good taste in conduct ar appearance. The sentence refers to a specific time when Judd’s behavior was diferent from his typical, or usual, behavior, which includes being reserved in social situations. The term “diplomat " might make sense in the first blank, because someone who is reserved in social gatherings might be escribed as being poised or tactful, but the term “decorum” does not logically complete the sentence. If Judd is usually reserved, polsed, and tactful, speaking and acting with decorum, or exhibiting appropriate conduct and good taste, would not be uncharacteristic FromSAT Online Course i CUUS ASIN ‘SAT 2006 May Answers and Explanations tothe Most pollticians find television ~ plays an essential role in propagating their ideas. of their messages: the medium © crucial. . renunciation V1) indispensable . . dissemination (©) detrimental .. communication (©) tangent. . interpretation (© relevant legislation ANSWERS AND EXPLANATIONS Explanation for Correct Answer B Choice (B) is correct. “Indispensable” means essential, or absolutely necessary, “Dissemination” means distribution. The structure of the sentence suggests that the text after the colon reiterates or further explains the text in the first half of the sentence, The second half of the sentence states that television ("the medium”) plays. fan essential role in propagating, or spreading, most politicians’ ideas. Therefore, it would make sense to claim in the first part of the sentence that most pollticians find television to be indispensable, or absolutely necessary, to the dissemination, or distribution, of their messages. Explanation for Incorrect Answer A: CChoice (A) is incorrect. “Crucial” means important or essential. To renounce means to ive up, refuse, or resign by formal declaration. The structure of the sentence suggests that the text after the colon reiterates or further explains the text in the frst half of the sentence. The second half of the sentence states that television ("the medium”) plays: ‘an essential role In propagating, oF spreading, mast poltcians’ Ideas; therefore, the first half of the sentence should express a similar concept. The term “renunciation” refers to giving up or refusing something, rather than distributing or spreading it, so the idea that politicians consider television to be crucial, or important, to the renunciation of their messages is the opposite of the idea expressed in the second half sentence, Explanation for Incorrect Answer C Choice (C) is incorrect. “Detrimental” means harmful ar damaging. “Communication” is 2 process of transmitting or expressing information, The structure of the sentence suggests that the text after the colon reiterates or further explains the text in the first the sentence states that television ("the mecium") plays an essential role in propagating, or spreading, most politicians’ ideas; therefore, the first half of the sentence should express a similar cancept. The iea that half ofthe sentence. The second half of ‘most politicians find television to be detrimental, or harmful, © the communication, or expression, of their messages is the opposite of the idea expressed in the second half 0 FromSAT Online Course Sih CUUS AB ‘SAT 2006 May Answers and Explanations f the ser Explanation for Incorrect Answer Dz Choice (D) Is Incorrect, “Tangental” means having litle relevance to something. To Interpret means to explain or understand the meaning of something. The structure of the sentence suggests that the text after the colon reiterates or further explains the text in the first half of the sentence. The second half of the sentence states that television ("the medium”) plays an essential role in propagating, or spreading, most politicians” ideas; therefore, the first half of the sentence should express a similar concept. The idea that most poltcians find television to be tangential, or to have litle relevance to, the "Interpretation", or understanding, of thelr messages Is not similar to the idea expressed in the second half of the sentence. Explanation for Incorrect Answer E Choice (E) Is incorrect. "Relevant" means relating or having a significant connection to something. “Legislation” is the act of making or enforcing laws. The structure of the sentence suggests that t 1 text after the colon reiterates or further explains the text the frst nalf of the sentence, The second half of the sentence states that television (the medium") plays an essential role in propagating, or spreading, most politicians’ Ideas; therefore, the first half of the sentence should express a similar concept. Since the term “lesislation” refers specificaly to the making and enforcing of laws, it does not make sense to say that most poiltcians find televislon to be relevant to the “legislation of their messages"—the sentence does not state that the politicians’ messages concern anything that would require legislation, and it would net be the message itself that would require legislation. Jazz musician Benny Carter, known for his sophisticated compositions and arrangements, was also 2 virtuoso alto saxophone player. Fellow musicians tine frequently cited Carter's groundbreaking improvisational 5 style, which avoided the expected run up and down the chord changes and instead spread out phrase fragments over the chord progression, In 1934, just six years after his first recording, Carter played at the opening of the Apollo Theater in Harlem and then led the first interracial big band 49 to tour Europe. In the ensuing quarter century, Carter wrote music for film and television in Hollywood, where he played 2 key role in the merger of the trade unions of Black musicians and White musicians. ‘The author suggests that Carter was not only an outstanding jazz composer but al FromSAT Online Course Sih CUUS HA ‘SAT 2006 May Answers and Explanations (®) a talented composer of classical musle (©) a highty imitative performer (©) an opponent of organized labor $10) an effective participant in social change (© an astute critic of work ANSWERS AND EXPLANATIONS: Explanation for Correct Answer D : Choice (D) is correct, While most of the passage focuses on Carter's musical achievements, it does also mention that Carter “led the first interracial big band to tour Europe." The author goes on to say that Carter “played a key role in the merger of trade unions of Black musicians and White musicians.” Carter played an important part in merging, or bringing together, two groups that were, during that time period, often segregated or separate, so the euthor suggests that Carter was an effective participant in social change. Explanation for Incorrect Answer A: Choice (A) is incorrect. The passage only makes reference to Carter as a jazz musician and a “virtuoso,” or highly skilled, saxophone player. The passage goes on to describe Carter's accomplishments and unique skills related to jazz music. Although itis possible that Carter may have written classical music (for example, “for film and television in Hollywood"), classical music is nat spectically mentioned in any part of the passage. Explanation for Incorrect Answer B Choice (B) is incorrect. The author of the passage refers to Carter's “groundbreaking Improvisational style,” To be improvisational, or to Improvise, means to compose or perform suddenly or unexpectedly. Since Carter was skilled at improvising, he often “avoided the expected run up and down the chord changes” that were most ikely @ ‘major part of other jazz musicians’ performances and “instead spread out phrase fragments over the chord progression.” Rather than suggesting that Carter was @ highly imitative performer, or that he imitated the technique of ather musicians, the author suggests that Carter's style was somewhat unusual. Explanation for Incorrect Answer C: Choice (C) is incorrect. While most of the passage focuses on Carter's musical achievements, it does also mention that Carter "played a key role in the merger of trade unions of Black musicians and White musicians.” The author suggests that Carter FromSAT Online Course CUUS WABI ‘SAT 2006 May Answers and Explanations played an important part in merging, or bringing together, organiznd labor groups, not that Carter was en opponent of organized labor, Explanation for Incorrect Answer E: Choice (F) Is incorrect. The passage does discuss Carter's "fellow musicians" recognition of Carter's “groundbreaking improvisational style,” Carter's leading of “the first interracial band to tour Europe,” and Carter’s "key role in the merger of the trade Unions of Black musicians and White musicians"), but the author does not address n that Carter's reaction to or erticism of other musicians’ work. There is no suggest Carter was an astute eric ‘The passage supports which of the following statements about Carter as a saxophone player? ©) He impressed other musicians with his technique (©) He was less celebrated as an instumentalist than as a bandleader. (©) He did not perform in pubic after the 1930s, (©) He confused erties with his unusual technique (© He preferred to perform in local venues, ANSWERS AND EXPLANATIONS Explanation for Correct Answer A: Cchoice (A) is correct. The passage refers to Carter as a “virtuoso,” or highly skilled, saxophone player whose “groundbreaking” technique involved avoiding "the expected run up and down the chore changes” and “instead [spreacing] out phrase fragments over the chord progression.” Carter's technique likely impressed ether musicians, because the passage states that his “fellow musicians” often cited Carter's style—that is, they praised and imitated it Explanation for Incorrect Answer B Choice (B) Is incorrect: The passage describes in detail Carter's technique and “groundbreaking improvisational style" as a saxophone player and mentions that Carter “led the first interracial big band to tour Europe.” Nothing in the passage suggests that Carter was less celebrated as an instrumentalist than as @ bandleader; in fact, because the passage gives such a detailed description of Carter's highly praised style and technique in playing 1 saxophone and only briefly mentions his role as a bandleader, fone might infer that Carter was actually more celebrated as an instrumentalist than as 2 bandleader. Explanation for Incorrect Answer C From SAT Online Co urse i CLUS ABI ‘SAT 2006 May Answers and Explanations Choice (C) is incorrect, The passage states that "In 1934,..Carter played at the opening {Apollo Theater in Harlem and then led the first interracial big band to tour Europe.” There is no mention of Carter's later performances, but its logical to assume that he continued to play his music for audiences. The passage does not support the statement that Carter did not perform in public after the 1930s. Explanation for Incorrect Answer D : Choice (D) is incorrect. The passage states that other musicians cited, or praised and Imitated, Carter’s “groundbreaking improvisational” style and technique, but it does not address crits! reaction to Carter's technique. The logical assumption would be that critics were as Impressed with Carter's technique as other musicians wer Explanation for Incorrect Answer E: Choice (F) is incorrect. Lines 8-11 Ingicate that Carter performed and wrote music in 2 variety of locations ("played at..the Apollo Theater in Harlem,” "led the first interracial big band to tour Europe,” *wrate music for lm and television in Hollywood"), but the passage does not address Carters preference for performing in any particular type of venue, local or otherwise, David Stahle is an expert on the imprints that climate, fire, and pestilence leave in a tree’s growth rings. His pulse rises when he spots a stand of beat-up tine old trees because they have stories to tell. For reasons 5 not fully understood, such trees, growing under harsh conditions, often live longer and are more sensitive to fluctuations in weather than healthier trees growing on better land. Recently, Stahle concluded from rings in bald cypress trees that a severe drought struck Roanoke 10 Island, North Carolina, in the late sixteenth century This might explain why the so-called Lost Colony of Roanoke died out by 1590. ‘The reference to Stahle’s “pulse” (line 3) serves to indicate his level of (8) vitality ©) frustration (©) anger ©) playfulness WO exctement ANSWERS AND EXPLANATIONS FromSAT Online Course Sith CUUS WAIN ‘SAT 2006 May Answers and Explanations Explanation for Correct Anwar E : Choice (F) is correct. The passage states that when Stahle sees "a stand of beat-up old trees,” his "pulse rises" they can provide him with information related to his studies on the effects of climate, fire, and pestilence on a tree's growth rings. His pulse likely rises because of his heart beats faster. Stahle knows such trees have “stories total his excitement to ciscover groups of trees he can study and learn from, Explanation for Incorrect Answer A: Choice (A) is incorrect. In this context, “vitality” refers to liveliness. Although vitality might be a part of Stahle’s personality, a rising pulse is not necessarily an indication of liveliness. Rather, its likely that Stahle’s pulse rises—that is, his heart beats faster—when he sees “a stand of beat-up old trees” because he is excited to study and leara from them. Explanation for Incorrect Answer B Choice (B) is incorrect. “Frustration” is a deep sense of dissatisfaction and insecurity caused by unresolved problems or unfulfilled needs. The passage states that Stahle is “an expert” on the effects of climate, fire, and pestilence on a tree's growth rings; there Is no reason to believe that seeing “a stand of beat-up old trees" would cause Stahle to feel frustrated. On the contrary, it Is most likely that his pulse rises—that Is, his heart beats faster—when he sees such trees because he knows they have “stories to tell” and Is excited to study and learn from them. Explanation for Incorrect Answer C: Choice (C) is incorrect, The passage states that when Stahle sees "a stand of beat-up old trees," his "pulse rises"—his heart beats faster. Stahle knows such trees have “stories to tell"—they can provide him with information related to his studies on the effects of climate, fire, ané pestilence on a tree's growth rings. Although anger can cause one’s pulse to rise, there Is no reason to believe that discovering trees would cause Stable to feel angry. On the contrary, itis most likely that his pulse rises because he Is excited to study and learn from the trees, Explanation for Incorrect Answer Dt Choice (D) is incorrect. The passage states that Stahle is "en expert” on the effects of mate, fire, and pestilence on 2 tree's growth rings and that his “pulse rises'—his heart beats faster—when he sees "a stand of beat-up old trees” because he knows the trees have "stories to tell." It is somewhat illogical to suggest that Stahle's reaction Indicates his level of playfulness, since he likely takes his study of trees seriously Rather, Stanle's rising pulse most likaly indicates that he is excited to study and learn from the trees he discovers. FromSAT Online Course CUUS WABI ‘SAT 2006 May Answers and Explanations ‘The statement in lines 11-12 ("This . . . 1590") serves primarily to ©) advocate a cause W®) offera theory (©) question a beliet (©) modity a cleim (©) predict an outcome ANSWERS AND EXPLANATIONS Explanation for Correct Answer Choice (B) is correct. In lines 9-10, the author of the passage states that, alter studying the rings in bald cypress trees, Stahle concluded that "a severe drought struck Roanoke Island..in the late sixteenth century." The author then suggests that this severe drought may have caused the Lost Colony of Roanoke to dle out, The author Is Utllzing Stahle's findings to offer a theory for the failure of the Roanoke colony. Explanation for Incorrect Answer At Cchoice (A) is incorrect. In lines 9-10, the author of the passage states that, after studying the rings In bald cypress trees, Stahle concluded that "a severe drought struck Roanoke Island..in the late sixteenth century." Based on Stahle’s conclusion, the fen suggests that this severe drought may have caused the Lost Colony of Roanoke to die aut by 1590. This suggestion does nat indicate that the author advocates, or supports, any particular cause—the author merely offers a possible explanation for something author Explanation for Incorrect Answer C Choice (C) is incorrect. In lines 9-10, the authar of the passage states that, after studying the rings in bald cypress trees, Stahle concluded that "a severe drought struck Roanoke Island..in the late sixteenth century.” Based on Stahle’s conclusion, the fen suggests that this severe drought may have caused the Lost Colony of Roanoke to die aut by 1590. This suggestion does not question a belief—elther Stahle’s belief that a drought struck Reanoke Island or any other bellef about the fallure of Roanoke. Rather, the author merely offers a possible explanation for something author Explanation for Incorrect Answer D: Choice (D) is incorrect. In nes 9-10, the author of the passage states that, after studying the rings In bald cypress trees, Stahle concluded that "a severe drought struck Roanoke Island..in the late sixteenth century." The author then suggests that this From SAT Online Co urse i CLUS ABI ‘SAT 2006 May Answers and Explanations severe drought may have caused the Lost Colony of Roanoke to die out by 1590. The F does not make any specifc claims regarding {allure of Roanoke, so he or she is not modifying a claim in the last sentence. Rather, the author is merely offering @ possible explanation for something. Explanation for Incorrect Answer E+ Choice (F) Is incorrect. In Ines 9-10, the author of the passage states that, after studying the rings in bald cypress trees, Stahle concluded that "a severe drought struck Roanoke Island..in the late sixteenth century.” Based on Stahle's conclusion, the author then suggests that this "severe drought” may have caused the Lost Colony of Roanoke ta die out by 1599. The author is not making a prediction; the outcome of the Lost Colony of Roanoke is already known—the colony died out. The author is merely offering 2 possible explanation for what may have caused this outcome, The frst of the following excerpts 's adapted trom » 1996 essay about an exhibit of paintings by the Dutch artist Johannes Vermeer (1632-1675); the second Is from an autobiography written in 1962. Passage 1 ‘Anyone who has read Edward Snow’s highly personal and poetic Study of Vermeeris unlikely to be able to meet the gaze of a young woman in The Girl with the Peart tine Earring without feeling something of the confusion and 5 complicity he so eloquently describes. “To meet this young gitt’s gaze is to be implicated in its urgency,” Snow writes, “It is me at whom she gazes, with real, unguarded human emotions, and with an intensity that demands something just as real and human in return.” Snow describes the girl's 40 gaze as a stew of unresolved contradictions, which only leaves the captivated viewer all the more guilty and confused. Surprise, bewilderment, yearning, acceptance, understanding —in sequence, or all at once—her glance seers to convey all of these. “Suddenly, the most personal 15 responses, drawn from the most private, well-protected regions of the self, are required.” My responses to this and other Vermeer paintings were rarely as personal and passionate as Snow's. To his private testimony regarding The Gir with the Pear! Earring, 1 20 would only add that a great deal of whatever impact it has on the viewer seems to me to derive from very specific, very visible painterly techniques. The light-colored spots FromSAT Online Course CUUS WABI ‘SAT 2006 May Answers and Explanations that appear to round her lip; the shift from light paint to darker that forms the ridge below her nose; the dots of 25 white on her pupils that make her eyeballs seem to bulge; the brusque, Cézanne-like rounding of her turban by shifting the blue strokes to black—all of these are as vivid as fingerprints, evidence of Vermeer’s presence, of his attempts (as Snow put it) to nurse this face into being 30 Today the entire painted surface of this work is spider- webbed with a fine network of cracks—what art critics call cracquelure—which makes the ability of this imaginary gitt to reach out and disturb us from her paint-created, 330-year-old flatness all the more astonishing. Passage 2 35 There was an empty room at the top of our house and on the wall, between the fireplace and the window, almost hidden in the gloom, hung a small portrait in olls, Dim and yellowed by time, that picture showed the head of a man of strange appearance, for he had long hair and below his 49 pointed beard was some kind of wide collar brim. His face was melancholy and yet it always seemed to me that there was a sneer upon those bearded lips. Children rarely look up when they are playing with toys on the floor, for they are too busy with their own. 45 imaginings, but one day, when 1 had strayed into that vacant room and was amusing myself, I happened to glance toward the portrait and saw, to my astonishment, that the eyes of the man were watching me. I tumed away, then looked up, and once more his eyes met mine. I got to my 50 feet and, as I did so, those eyes turned to watch every movement I made. I backed toward the door and that look still followed me, half-mournful, half-accusing. T did not tell anyone what had happened, but I did not venture into the room again for more than a few seconds at 55a time, except when I was with grown-ups, Emboldened by their company, I learned to play a fearful but exciting game. I moved slowly here and there about the room, knowing that wherever I went, those eyes followed me. I pretended not to look, I talked with my elders, then swung 60 around sharply. I was still being watched. The searching gaz was always the same, so melancholy and accusing that T began to feel a sense of guilt, began to wonder what evil I FromSAT Online Course it CUUS WoAB i ‘SAT 2006 May Answers and Explanations could have committed. Guilt and fear mingled in my mind and I was certain that the stranger was treacherous, that his 65 anger could be terrible when it was aroused. Sometimes greatly daring, I crept up the stairs and opened the door very quietly, very quickly, and peeped in. Every time I did so, his eyes were looking straight at me, Which of the following statements best characterizss the individual perspectives presented in the two passages? (09 The first passage presents a critical perspective, whereas the second presents the view of an uneducated art lover (©) The first passage offers the perspective of art historians, whereas the second offers that of an art student, (©) The first passage offers the perspective of adult art collectors, whereas the second contains the views of a child prodigy. WO) The first passage offers the views of two adults, whereas the second presents @ chiles perspective (© The first passage offers the views of two successful artists, whereas the second presents the view of a failee artist: ANSWERS AND EXPLANATIONS: Explanation for Correct Answer D : Choice (D) is correct. In the first passage, the author describes Edward Snow's, “personal and poetic” reaction to Vermeer’s painting as well as his or her own mare ‘methodical and analytical explanation of Vermeer’s artistic technique, Both of the descriptions in Passage 1 are from an adult perspective. In the secand passage, the narrator describes his or her reaction to @ painting when the narrator was a child, “playing with toys on the floor. Explanation for Incorrect Answer A: Choice (A) is incorrect. The first passage does offer a critical perspective of Vermeer’s painting, but it also presents Snow's "personal and poetic” reaction to Vermeer’s The Girt with the Pear! Earring. The narrator of the secand passage, however, describes a child's reaction to 2 portrait. While the perspective may be naive, nothing in the passage incicates that the chilé is an “uneducated art lover.” Explanation for Incorrect Answer Choice (B) is incorrect. Act historians are not discussed in Passage 4. Although one right assume that Edward Snow—author of Study of Varmeer—is an a author of the passage does not indicate that he is an art historian. Further, the author istorian, the FromSAT Online Course Sih CUUS WA ‘SAT 2006 May Answers and Explanations of the passage does not identity himself or herself as an art historian. Passage 2 does rot present the perspective of an art student, Tt tells the story of the author's recollection of his or her childhood, specifically, his or her response after looking at a portrait hanging In the room where the author happened to be playing. Nothing In the passage incicates that the child is an art student. Explanation for Incorrect Answer Cz Choice (C) is incorrect. The author of the first passage and the author he or she quotes are adults and may well be art collectors, too, But nothing in the text indicates that collecting artis their profession or thelr hobby. Instead, the passage meraly incicates what Edward Snow and the narrator have said about Vermeer’s painting, The second passage describes a chil’s response to a portrait, but nothing in the passage indicates that the child is @ “prodigy,” or genius. Explanation for Incorrect Answer E: CChoice (E) Is incorrect. The first passage presents two different reactions to @ painting by Vermeer. Both Edward Snow and the narrate own, but the passage does not tell us whether they are successful artists. Nothing in the second passage indicates that the narrator is a "alled artis," either. The passage merely describes the narrator's reaction to a portrait when he or she was still child F may have made paintings of thelr ‘The first paragraph of Passage 1 (lines 1-16) primarily serves to (9 indicate how critics have tended to see Vermeer's work in the past (©) outtine the major events In the ife of Vermeer (©) demonstrate the modern qualities of Vermeer’s The Git with the Pear! faring (©) highlight the significance of Snow's Study of Vermeer W) discuss Snow's response to a painting by Vermeer ANSWERS AND EXPLANATIONS: Explanation for Correct Answer E Choice (E) Is correct, The first paragraph primarily describes Snow’s “personel and poetic” response to Vermeer’s The Girl with the Pear! Earring. The author emphasizes Snow's emotional and contradictory reactions toward the painting (*Surprise, bewilderment, yearning, acceptance, understanding." Explanation for Incorrect Answer A Choice (A) is incorrect. The first paragraph primarily ciscusses how onecrtic, Edward FromSAT Online Course Siti CUUS WABI ‘SAT 2006 May Answers and Explanations Snow, describes Vermeer The Gir! with the Pearl Earring. does nat indicate how critics in general have respondes to Vermeer’s work. Explanation for Incorrect Answer Choice (B) is Incorrect. The first paragraph does not provide an outline of Vermeer's Ife t primarily describes Snow's "personal and poetic" response to Vermeer’s painting ‘The author emphasizes Snow's emotional and contradictory reactions "Surprise, bewilderment, yearning, acceptance, understanding..”) toward one painting in particular. Explanation for Incorrect Answer C Choice (C) is Incorrect. The first paragraph primarily cescribes how a critic, Edward ‘Snow, responds to one of Vermeer’s paintings in the Study of Vermeer. Nothing In the paragraph indicates that Snow's “highly personal and poetic” study is illustrative of Vermeer's "modern qualities” in The Girl with the Pear Earring. Explanation for Incorrect Answer D : CChoice (D) is incorrect. The first paragraph highlights Snow's "personal and poetic” reading of Vermeer's The Gir with the Pearl Earring. brings to light the variaus contradictory emotions thet Vermeer's painting elicits in Snow. The author refrains from making a judgment about the impact of Snow’s work; the reader cannot tell from reading the first paragraph if Snow's study has made a significant impact on Vermeer criticism or not. According to the author of Passage 1, the “confusion” mentioned in line 4 refers to the (8) author's initial misunderstanding of Vermeer's painting (©) arcane language Snow uses in his study of Vermeer (©) facial expression of the figure depicted in the painting 1) complex emotions experienced by someone viewing the Vermeer painting (© Intricate arguments a critic has made about Vermeer’s genius ANSWERS AND EXPLANATIONS: Explanation for Correct Answer D : Choice (D) is correct, When the author uses the ward “confusion” he or she refers to the bundle of contradictory emotions that Snow feels when he looks at Vermeer’s The Girl with the Pear! Earring—a combination of “surprise, bewikierment, yearning, acceptance, [and] understanding.” The author suggests that anyone who reads Snow's FromSAT Online Course Sith CUUS WAIN ‘SAT 2006 May Answers and Explanations study wil arobably have similar reactions upon seeing the painting Explanation for Incorrect Answer A: CChoice (A) Is incorrect: The author uses the word “confusion” to indicate the contradictory nature of Snow's reaction to The Girt with the Pear! Earring. Snow's contradictory emotions mirror the feelings that Snow sees in the gir's ga ("stew of Unresolved contradictions”); they do not represent @ misunderstanding of the painting Explanation for Incorrect Answer Choice (B) is incorrect. According to the author, the word “confusion” refers to the contradictory and complex emotions Snow feels when he meets t 'e young Girl in Vermeer’s The Git with the Peart Earring. “Arcane” means mysterious and secret ‘While Snow's reactions to Vermeer’s painting are contradictory and complicit, they are not described in language that is arcane or mysterious. we gam oft Explanation for Incorrect Answer C Choice (C) is incorrect. The author indicates that Snow's reaction to Vermear’s The Gir! With the Pearl Earring 's one of “confusion and complicity.” This description refers to the bundle of contradictory emotions that Snow feels upon looking at the gir's face. While this confusion is ellcted by the gir’s gaze, the author refers to Snow's description of his ‘own emotions, not to the figure’s expressions Explanation for Incorrect Answer E Choice (E) Is incorrect. The authar of Passage 1 does not state that Snow's Study of Vermeer included any “intricate arguments” concerning Vermeer’s genius. Instead, the author describes Snow's "personal and passionate" reaction to The Girl with the Pear! Earring—2a reaction that included contradictory emotions, or confusion. In context, which of the following observations from Passage 1 most nearly parallels an observation the narrator of Passage 2 might make? W1)"Ta mest this young girl's gaz Is to be implicated in is urgency’ (lines 5-6) (®)-my responses to this and other Vermeer paintings were rarely as personal and passionate as Snow's” (lines 17-18) (Clrwhatever impact it has on the viewer seems to m derive from very specific, very visible painterly techniques” (lines 20-22) ("all of these are as vivid as fingerprints, evidence of Vermeer’s presence” (lines 27-28) (©-Today the entire painted surface of this work is spider-webbed with @ fine From SAT Online Co urse i CLUS ABI ‘SAT 2006 May Answers and Explanations network of cracks” (lines 30-31) ANSWERS AND EXPLANATIONS: Explanation for Correct Answer At Choice (A) is correct. In ines 5-6, the word “urgency” refers to the rush of emotions the viewer feels when his or her gaz meets that of the young girl in the painting, The connection between them seems immediate, or urgent; the author feels implicated,” or Involved, in the action of the painting. The narrator of Passage 2 had a similar response to a portrait when he or she first looked at it as a child: the reaction seemed real, immediate, and urgent ("I turned away, then looked up, and once more his eyes met mine. I got to my feet and. “r began to feel a sense of guilt”) ‘ose eyes turned to watch every movement I made,” Explanation for Incorrect Answer B Choice (6) is incorrect, Lines 27-18 reflect the point of view of the author of the. passage, who responds to Vermeer's painting in the opposite way that Snow does: the ‘author responds in mostly objective and dispassionate ways. The narrator of Passage 2 was hardly objective and dispassionate about he portrait he or she encountered as a child. Rather, he or she reacted in “astonishment” and began to feel frightened when the man's eyes appeared to follow him or her throughout the room, Explanation for Incorrect Answer C1 Choice (C) is incorrect. Lines 20-22 refer to the author's objectivity as he or she observes Vermeers painting; rather than describing his or her emotional state, the author explains the complexity of the gaze by describing Vermeer’s "painterly techniques.” In contrast, the narrator of Passage 2 adopts a child's perspective and scribes the powerful effects of the portrait—astonishment, fear, guit—in highly personal terms. Explanation for Incorrect Answer D: CChoice (D) is Incorrect, Lines 27-28 express the author's objective ane scholarly observation of Vermeer’s painting. The author looks for Yevidence” of style ("Vermeer’s presence”) and painterly techniques rather than for the painting’s emotional impact. In contrast, the narrator of Passage 2 adopts a chile’s perspective and describes the effects of the portrait on him or her in highly personal and emotional terms: astonishment, fear, guilt Explanation for Incorrect Answer E: Choice (F) is incorrect. In lines 30-31 the author describes what the surface or texture of Vermeer’s painting looks lke today. The description is given in a detached, objective FromSAT Online Course i CUUS ASIN ‘SAT 2006 May Answers and Explanations ‘manner, using the specialized art term “cracquelure.” In contrast, the narrator's observations in Passage 2 are mostly presented from the perspective of a child, who experiences the figure in the portrait as life-like ("to my astonishment..the eyes of the rman were watching me"). Nowhere In Passage 2 do we find the type of technical and detached observations that we find in lines 30-31. ‘The second paragraph of Passage 1 (lines 17-29) primarily serves to (A) itustrate the narrator's emotional response to the painting © outine the different ways erties have viewed Vermeer (©) compare the artistic achievement of Cézanne and Vermeer ©) aiscuss how Vermeer achieved specific effects in his painting © descrive the materials Vermeer used in his painting ANSWERS AND EXPLANATIONS: Explanation for Correct Answer D : Choice (D) is correct. In ines 17-29, the author primarlly describes Vermeer’s “painterly techniques,” or style. Specifically, the author describes the effects of "ight- colored spots” on the figure’s lip, the shift from light to dark paint, the use of “dots of white" paint forthe pupils, and the shift from blue to black brush strokes. Explanation for Incorrect Answer At Choice (A) is incorrect. In the second paragraph of Passage 1, the author does nat display any emotional response to Vermeer’s painting. Rather, the author provides a detailed description of Vermeer’s artistic techniques, such as the use of color, light, and brush strokes Explanation for Incorrect Answer B CChoice (B) is incorrect, Lines 17-29 do not describe “eifferent ways crtics have viewed Vermeer.” Rather, they describe the author's own observ techniques*—the use of color, light, and brush strokes. n of Vermeer’s "painterly Explanation for Incorrect Answer C: Choice (C) is incorrect. The author's description of Vermeer's *Cézanne-lke rounding of the gir's) turban” constitutes just ane aspect of Vermeer’s artistic achievement. In other words, the author is likening one detall of Vermeer’s painting to Cézanne, not camparing the two artists’ accomplishments or careers Explanation for Incorrect Answer FromSAT Online Course Sith CUUS WAIN ‘SAT 2006 May Answers and Explanations Choice (E) is incorrect. The author does not describe the materials, but rather the “painterly techniques” that Vermeer used to paint the figure InThe Gir! with the Pear! Farring. The author describes such artistic techniques as the use of color, light, and brush strokes. In relation to the reaction of the narrator of Passage 2 to the painting, the reaction of the author of Passage 1 can best be described as (§) more emotional ¥®) more analytical (©) more fearful ©) ess idealistic © ess complimentary ANSWERS AND EXPLANATIONS: Explanation for Correct Answer Choice (B) Is correct. The narrator of Passage 2 describes his or her emotional reaction te the portrait (“T began to feela sense of guilt, began to wonder what evil I could have committed"), and emphasizes the sensation of being followed by the man’s “half- mournful, hatf-accusing” look. In contrast, the author of Passage 1 gives us = more analytical or scholarly description of Vermeer’s “paint rly techniques” and describes the artist's use of color, brush strokes, and light effects. Explanation for Incorrect Answer A Choice (A) is incorrect, The narrator of Passage 2 emphasizes the various emations (Castonishment,” "ear," “guilt that he or she felt upon looking at the portrait as & child. In contrast, the author of Passage 1 primarily descrives Vermeer’ artistic effects cr “painterly techniques” in a more detached, scholarly way. So it isthe narrator of Passage 2 who can be described as having a "more emotional!” reaction, Explanation for Incorrect Answer C: Choice (C) is incorrect. The narrator af Passage 2 describes the fear he or she experienced as a child when glancing at the portrait and observing the man’s seemingly roving eyes: “I did not venture into the room again for more than a few seconds at @ time.” and “Guilt and fear mingled in my mind and I was certain that the stranger was treacherous...” In contrast, the author of Passage 1 does not cisplay fear when he or she looks at Vermeers painting, but, rather, describes the painting in more analytical, scholarly ways and emphasizes Vermeer’s “painterly techniques,” such as color, brush strokes, and light effects From SAT Online Co urse i CLUS ABI ‘SAT 2006 May Answers and Explanations Explanation for Incorrect Answer D: Choice (D) is incorrect. To be idealistic isto form and maintain ideals, or standards of beauty, perfection, or excellence. The reaction of the narrator of Passage 2 is not Idealistic; the child does not consider the portrait in terms of standards of beauty or excellence, but rather reacts to it emotionally. The portrait makes the chilé fearful (Guilt and fear mingled in my mind.."). In contrast, the author of Passage 1 does consider the excellence of Vermeer‘s painterly techniques, s0 his or her reaction might be described as more, not less, Idealist, Explanation for Incorrect Answer E: CChoice (E) Is incorrect. The author of Passage 1 might be said to be “complimentary” about The Gir with the Pearl Earring. But the narrator of Passage 2 is completely shaken up by the feelings of fear and guilt he or she experiences upon looking at the man’s eyes; the narrator doas not describe the portrait in complimentary ways. So if anything, the author of Passage 1 Is more, nat Jess, complimentary than the narrator of Passage 2. ‘The first paragraph of Passage 2 (lines 35-42) Is primarily concerned with W/) providing a physical description of the portrait (®) showing the child's growing fear of being watched (©) revealing how the chilé overcame an irrational disike of the portrait (©) discussing some techniques used in the painting © explaining the significance of the man in the portrait ANSWERS AND EXPLANATIONS: Explanation for Correct Answer A: CChoice (A) Is correct, The first paragraph in Passage 2 primarily describes the physical ibutes of the man in the portrait: long hair, pointed beard, and what seems to the narrator like a "sneer upon those bearded lips Explanation for Incorrect Answer B Choice (B) Is incorrect: There is nothing In the fist paragraph of Passage 2 that shows “the child's growing fear of being watched.” Lines 35-42 primarily describe the room and the physical attributes of the man in the portrait. While the man in the portrait is, described as looking “melancholy” and having a “sneer” on his lips, these descriptions. are not indicative of the child's fear. Explanation for Incorrect Answer C+ From SAT Online Co urse i CLUS ABI ‘SAT 2006 May Answers and Explanations Choice (C) is incorrect. The first paragraph in Passage 2 is primarily a description of the roam and the portrait. Since the narrator does not discuss the child's fearful reaction to the portrait until the third paragraph, the first paragraph does not reveal "how the child overcame an Irrational dislike ofthe portrait.” In fact, there Is no information at all in the passage about how or ifthe child overcomes his or her fears. Explanation for Incorrect Answer D : CChoice (0) is incorrect. The narrator does not discuss the painter's techniques in the first paragraph of Passage 2. The narrator merely describes the room in which the portrat is found ("between the fireplace and the window, almost hidden In the gloom") and the physical attributes of the man in the portrait, including a description of his halt, beard, and facial expression. While the narrator states that the portrait is “dim and yellowed by time," ths is not a reference to the artist's technique, Rather, its Suggestive of what happens to 2 painting with the passage of time Explanation for Incorrect Answer E: Choice (E) is incorrect. The “significance” of the man represented in the portrait is not Clear in the first paragraph of Passage 2, The narrator merely offers a physical description of both the room and the portrait. The narrator does not tell us in what way the man in the portrat is significant ‘The reaction of the child in Passage 2 to the painting (lines 43-52) was primarily one of (8) abject terror (©) overwhelming curiosity VO growing unease (©) cool incitference © unconcealed acmiration ANSWERS AND EXPLANATIONS Explanation for Correct Answer C : Choice (C) is correct, In lines 43-52, the narrator recounts the "growing unease” he or she experienced as a child. The child was innocently playing, "strayed into" the room where the portrait hung, became aware of the portrait, and subsequently became astonished by a particular aspect of the portrait—the man’s eyes. The chilé felt Increasingly uneasy until he or she finally "backed & 1¢ door,” still imagining that the man's "eyes turned to watch every movement.” ward Explanation for Incorrect Answer A FromSAT Online Course Sih CUUS HA ‘SAT 2006 May Answers and Explanations Choice (A) is incorrect. The description n lines 43-52 suggests that the child did not experience “abje: rror"—hopeless, intense fear— but rather a growing sense of Unease. The child merely noticed the portrait, became astonished by a particular aspect of (the man’s eyes), and then “backed toward the door” while imagining that the man’s eyes "stil olowed” him or her Explanation for Incorrect Answer Choice (B) is incorrect. Although the description in lines 43-52 suggests that the chil was astonished by and interested in the movement of the man’s eyes, "overwhelming Curiosity" was not his or her primary reaction. The child did nat ask how the man’s eyes happen to move, Instead, the child began to feel uneasy and eventually left the room, Imagining that the man’s eyes “stil followed” him or her. Explanation for Incorrect Answer D Choice (D) is incorrect. The reaction described in ines 43-52 is certainly not one of “cool indifference,” or nonchalance. Rather, the child felt “astonishment” or surprise at seeing the man’s eyes follow his or her movements, Explanation for Incorrect Answer E Choice (E) Is incorrect, Nothing in the description n lines 43-52 suggests that the child reacted to the portrait with “uncancealed admiration.” Rather, the child felt uneasy after noticing that the man’s eyes seemed to follow his or her movernents, and eventually the child "backed toward the door” Ifthe child had admired the painting, he or she likely would have stayed to continue looking at it ‘The narrator of Passage 2 looks to adults for (9) companionship (©) amusement VO) security (©) guidance © approval ANSWERS AND EXPLANATIONS Explanation for Correct Answer C CChoice (C) is correct. The narrator of Passage 2 suggests that as a child, it was only in the presence of adults that he ar she dared to enter the room where the portrait was hanging ("I did not venture into the room again..except when I was with grown-ups") ‘The child thus looked to adults for security and felt “emboldened by their company. FromSAT Online Course Sih CUUS HA ‘SAT 2006 May Answers and Explanations Explanation for Incorrect Answer A: Choice (A) is incorrect. The narrator of Passage 2 did not necessarily desire companionship from adults; he or she simply needed protection from the watchful eyes the man in the portralt, When the narrator “talked with [his or her] elgers,” he or she did so as part of the “fearful but exciting game” of 9 the man’s gaz. Explanation for Incorrect Answer Choice (B) Is incorrect. Although the narrator of Passage 2 claims to have played @ “fearful but exciting game" by pretending notto look at the portrait and talking to ‘grown-ups, the child was not primarily amused by adult company. It was the man in the portrait and not the adults with whom the chilé was concerned Explanation for Incorrect Answer Dt Choice (D) is incorrect. The narrator of Passage 2 did not talk to the grown-ups as a child in order to receive guidance, The reason he or she turned to the adults was to shield himself or herself from the menacing and accusing eyes of the man in the portrait ("I did not venture into the room..except when I was with grown-ups") and to test the tenacity of the man's gaz ("I pretended not to look, Ttalked with my elders, then swung around sharply. I was still being watched"), Explanation for Incorrect Answer E Choice (E) Is incorrect. We cannot tel from Passage 2 if the narrator sought approval from the elders. What the narrator tells us is that because he or she was frightened by the persistently “accusing” gaze of the man In the portralt, the child refused to go back In the room without the presence of adults (“I did not venture into the room .except when I was with grown-ups") ‘The narrator of Passage 2 experiences emotions that are most similar to those of the () “young woman” (line 3) W®) viewer" (line 11) (©) sman* (ine 38) (©) “grown-ups” (line $5) (© “stranger” (line 64) ANSWERS AND EXPLANATIONS Explanation for Correct Answer B FromSAT Online Course kh CUUS HERS, ‘SAT 2006 May Answers and Explanations Choice (B) is correct. The emotions experienced by the narrator of Passage 2 are much like those of the "viewer" Inline 11 of Passage 1. Bo: are “captivated” or excited by the gam of the figure before them. Both feel “guilty” because of the intense emotions that the gaz seems to reveal about each viewer, In Passage 1, the author states that the “stew of unresolved contradictions” leaves the “captivated viewer all the more guilty and confused.” The narrator of Passage 2 admits, “I began to feel a sense of guilt began to wonder what evil I could have committed. Gullt and fear mingled in my mind." Explanation for Incorrect Answer A : Choice (A) is incorrect, The narrator of Passage 2 does not feel emotions similar to the “young woman’ in Passage 1 (the ir! painted by Vermeer). The young woman in the painting elicits and appears to experience "a stew of unresolved contradictions” such as “surprise, bewilderment, yearning, acceptance, [and] understanding.” Tn contrast, the narrator of Passage 2 mostly experiences fear, guilt, and astonishment, which are not contradictory reactions. Explanation for Incorrect Answer C1 Choice (C) is incorrect. The narrator of Passage 2 feels astonishment or surprise, as well as fear and gult, but the "man" referred to In line 38 appears to be sad (*His face was melancholy”) and seems to sneer at or mock the onlooker (“it always seemed to ime that there was e sneer upon those bearded lips"). These two sets of reactions are very different from one another. Explanation for Incorrect Answer D: Choice (D) is incorrect. While the narrator of Passage 2 tells the reader what he or she felt as child in response to the portrait, the narrator does not reveal the emotions of, the "grown-ups" in the room. Nothing in the passage indicates how the adults react to the portrait, All the emotions described in the passage are attributed either to the child fr to the man depicted in the portrait. Explanation for Incorrect Answer E Choice (E) Is incorrect. From the point of view of the child, the "stranger" to whom the narrator refers in ine 64 appears angry: *[the strenger’s] anger could be terrible when it was aroused.” Anger is not an emotion the narrator of Passage 2 experiences, making It very unlikely that the narrator (as @ child) and the “strenger” have similar emotions In line 60, “searching” most nearly means FromSAT Online Course Sih CUUS HA ‘SAT 2006 May Answers and Explanations () hidden (©) beseeching © resourceful (©) forlorn © penetrating ANSWERS AND EXPLANATIONS Explanation for Correct Anwar E : CChoice (E) Is correct. When the narrator describes the man’s eyes as “searching,” he or she refers to the man’s seemingly penetrating or perceptive gaz. The man’s gaz Is considered by the child to have the power to see through or penetrate his or her most Inner thoughts: "I began to feel a sense of guilt, began to wonder what evil ! could have committed.” Explanation for Incorrect Answer At Choice (A) is incorrect. When something is hidden it cannat be seen; itis concealed or out of sight. In Passage 2 the gem of the man depicted In the portrat Is present at all times and is continuously visible ta the child. Thus, the word “searching” does not mean “hidden.” Explanation for Incorrect Answer B Choice (B) is incorrect, To beseech is to beg or plead for something. When the narrator Goscribes the effect of the man’s eyes as "searching gaze" that appears to accuse the narrator end to make him orher feel gulty and fearful, he or she implies that the man’s gam has power over the onlocker. Someane ar something with power wauld not beseech or beg. Explanation for Incorrect Answer C+ Choice (C) is incorrect, To be resourceful isto be Inventive or practical enough to be able to get what you want or need. According to the narrator the man's gam already seems to know the child's deep thoughts and appears to accuse the narrator of some evil deed. Therefore, the word “resourceful” is not the best substitution for “searching Explanation for Incorrect Answer D : Choice (D) is incorrect. To be forlorn means to be sad and lonely because of abandonment or loss, While the narrator mentions that the man's “searching gaz appears "melancholy," the narrator primarily perceives the man’s gaze asa relentless Invasion of his or her most Inner thoughts, Therefore, the word “forlorn is nx substitution for the ward “searching” in line 60. he best FromSAT Online Course CUUS WABI ‘SAT 2006 May Answers and Explanations ‘The descriptions of the paintings in both passages suggest that paintings () require critical understanding to be appreciated fuly (©) cannot withstand the test of ime ©) can have @ powerful impact on the lfe ofa viewer (©) need careful restoration to achieve thelr full power (2) may fil viewers with a feeling of nostalgia ANSWERS AND EXPLANATIONS Explanation for Correct Answer C: Choice (C) is correct. In the first passage, Snow's observ reveals that art has a powerful impact in the viewers life, For example, the painting elicits "responses, drawn from the mast private, well-protected regions of the seF.” Similarly, the narrator of Passage 2 Is profoundly affected by the portrait, The narrator describes how the man’s seemingly relentiess gaz arouses fears and feelings of guilt in the child (wonder what evil I could have committed”). The painting thus has an impact fon the child and elicits deep emotions that remain alive in the narrator's memory. of Vermeer’s painting Explanation for Incorrect Answer A: Choice (A) is incorrect. In Passage 1, we can see that while Snow appreciates the painting through an intimate connection with its subject, the author himself or herself does not need to feel such a connection to recognize the value of Vermeer's painting ‘This viewer's appreciation derives from a scholarly understanding of Vermeer's technique. There is no eviéence that the narrator of Passage 2 has a “critical”—or professionally evaluative —understanding of the portrait. Explanation for Incorrect Answer B CChoice (B) is incorrect, In Passage 1, the author alludes to the fact that even very old paintings (such as the "330-year-old" The Girl with the Pear! Earring) can "reach out ‘and disturb us"; they can, that \s, withstand the passage of time in their impact ang Influence on the viewer. In Passage 2, the narrator describes how the portrait—even though itis “dim anc yellowed by time*—affect narrator's memory. Bath narratives incicate that a painting's power can withstand the the viewer and remains in the test of time. Explanation for Incorrect Answer D: Choice (D) Is incorrect. The message in both passages |s that paintings have the power to affect us even when they are physically damaged. For example, even ifthe surface FromSAT Online Course Sih CUUS HA ‘SAT 2006 May Answers and Explanations lf Vermeer's painting is damaged by cracks or eracquelure, “the ability of this reach out and 6 Imaginary girl b us from her paint-created 330-year-old flamness is] allthe more astonishing.” As for the painting in Passage 2, even though the portrait Is “aim and yellowed by time" It maintains the power to startle the chlld: "to my astonishment..the eyes of the man were watching me.” Neither painting seems to have been restored, but both still have the power to affect the viewer Explanation fer Incorrect Answer E CChoice (E) Is incorrect, Neither the viewers of Vermeer in Passage 1 or the viewer of the portrait in Passage 2 are fesling nostalgia, homesickness, of longing for a return to the past while looking at paintings. They are feeling a sense of astonishment or wonderment at the powerful effects that the paintings have on them. Sections: Critical Reading Online - Practice Test #1 Canaries are often said to have voices because they make such sweet, harmonious sounds. (1 insistent (© unnatural WO melodious © inaudible © strident [ANSWERS AND EXPLANATIONS: Explanation for Correct Answer C : Choice (C) is correct, If something Is "melodious", it contains @ pleasing succession of sounds. The structure of the sentence indicates that the missing word will describe canaries as having volces that make “sweet, harmonious sounds," so the term “melodious” fits perfectly here Explanation for Incorrect Answer A: Choice (A) is incorrect. “Insistent” means repetitive and persistent. although the sound that canaries make could at times be considered insistent, or repetitive, the term does rot make sense in this context. The canaries’ voices are described as making “sweet, harmonious sounds“—not repetitve, persistent sounds, Explanation for Incorrect Answer arse i CLUS HAL From SAT Online Co ‘SAT 2006 May Answers and Explanations Choice (B) is incorrect. If something is “unnatural” It is considered to be contrived or artificial, The term does not make much sense in this context because the canaries’ voices are described as making "sweet, harmonious sounds” and not artificial or unnatural sounés. Explanation for Incorrect Answer D: Choice (D) is incorrect. “Inaudible” means Impossible to hear. The term makes no sense In this sentence because "sounds" are by definition auible and can be heard. Explanation for Incorrect Answer E Choice (E) Is incorrect, “Strident” means harsh or shri, The term makes no sense In the context of this sentence because the canaries’ voices are described as making “sweet, harmonious sounds,” not harsh, shrill sounds. ‘The ~---- plots of Agatha Christie's detective stories reflect the complexities hidden beneath the simple appearances of English vilage ie. VO intricate (©) aiscrete (©) straightforward (©) elementary © vacuous ANSWERS AND EXPLANATIONS Explanation for Correct Answer A : Choice (A) Is correct. “Intricate” means complex. Because Agatha Christie describes the “complexities” of English vilage life in her detective stories and not just the simple appearance, its logical that her plots would be characterised as intricate, or complex Explanation for Incorrect Answer Choice (B) Is incorrect, “Discrete” means consisting of unconnected parts, Solving the mystery in a detective story usually consists of figuring out the connection between seemingly unrelated parts. Therefore, it does not make sense to use the term “discrete” in this context, since the parts or clues do relate to one another—just not in aan obvious way. Explanation for Incorrect Answer C: Choice (C) Is incorrect, “Straightforward” means direct Agatha Christe’s stories detail the intricacies or complexities of village Ife. Therefore, it makes no sense to say that th urse CLUS HA From SAT Online Co ‘SAT 2006 May Answers and Explanations ese stories are straightforward, or direct. Explanation for Incorrect Answer D : Choice (D) Is Incorrect. “Elementary” means constituting the basic, essential, or fundamental part. Agatha Christie focuses on the complexities of English village life in her detective stories, not the simple—or basic—appearance. Therefore, it does not make sense to use the term “elementary” In this context, Explanation for Incorrect Answer E Choice (F) is incorrect. *Vacuous” means having no substance or meaning. Because Agatha Christie's stories are detective storles, they focus on deriving meaning from clues. Therefore, it does not make sense to use the term "vacuous" in this context. Government officials authorized —----- because they wanted to have a complete ‘enumeration of the country's population. (1 @ discontinuance (an appointment (©) a waiver 1) a census © ate ANSWERS AND EXPLANATIONS Explanation for Correct Answar D : Choice (D) is correct. A *census" Is a population count. Because the sentence indicates of the country’s population, it makes sense to use the term "census" in this context that government officials authorimd a complete “enumeration,” or cour Explanation for Incorrect Answer At Choice (A) Is incorrect. A discontinuance” is the termination of an action. Tt does nat ‘make much sense to say that government officials would have “authorized discontinuance” because they wanted an “enumeration,” or count, of the country's population. Explanation for Incorrect Answer B Choice (B) is incorrect. In this context, an "appointment" is the assignment of someone to fl an office or position. Although the government officials may have attained their positions by appointment, the structure of the sentence focuses on the country's population count. Therefore, the term “appointment” does not make sense In this conte FromSAT Online Course Sih CUUS AB ‘SAT 2006 May Answers and Explanations Explanation for Incorrect Answer C: Choice (C) Is incorrect, A "waiver" Is the act of Intentionally abandoning a known right. ‘The sentence does not focus an the rights of the country’s citizens, but on an “enumeration,” or count, of the country's population. Therefore, it does not make sense to use the term *walver” in this sentence. Explanation for Incorrect Answer E Choice (F) is incorrect. A “levy” is a tax. The term levy” does nat make sense in the context ofthis sentence; the government officials authoriaed a complete “enumeration,” or count, of the country’s population, not a tax. Even in her fiction writing, Denise Chavez functions as a kind of historian in that she ~~--- the real experiences of Hispanic women through her characters. ©) predicts (© detends A) chronictes ©) averts (© surmises ANSWERS AND EXPLANATIONS Explanation for Correct Answer C : Choice (C) is correct. To chronicle is to record or relate facts in the form of a historical record. Because Chavez writes about or records real experiences in the manner of a “historian,” the term “chronicles” makes sense used in this context. Explanation for Incorrect Answer At Choice (A) is incorrect. To predict Is to declare or guess in advance. The sentence Indicates that the author Chavez draws upon the real experiences of women In her writing, Chavez does not predict what these experiences will be, but rather gets ideas for her stories from events that have already occurred. Explanation for Incorrect Answer B Choice (B) is incorrect, “Defends” means sticks up for or protects against harm. There Is no indication in the sentence that Chavez needs to defend these women or their experiences, so the term “defends” does not logleally complete the sentence. From SAT Online Co arse i CLUS HAL ‘SAT 2006 May Answers and Explanations Explanation for Incorrect Answer D: Choice (D) is incorrect. “Averts” means turns away from or avoids. There is no Indication in the sentence that Chavez's turning away from these women’s experiences, rather, she is recording them. Explanation for Incorrect Answer E Choice (E) Is incorrect. To surmise is to guess. Although a reader of Chaver's stories may make a guess as to what will occur next as the plot progresses, there Is no Ingication that Chavez is guessing what will happen. Therefore, the term “surmises” does not logically complete the sentence, Fanatically committed to one political cause, Anderson was a an exclusively -, maintaining ~ outlook. © tbertine . . provincial WO) malot .. partisan (©) patriot... indulgent (©) maverick ..atruistc (© rebel various ANSWERS AND EXPLANATIONS Explanation for Correct Answer Br Choice (B) is correct. A “aalot” is someone marked by fervent partisanship to a person, a cause, or an ideal, To be "partisan" is to firmly adhere to a party, faction, cause, or person. Anderson's fanatical commitment to "politcal cause" can certainly be described as fervent partisanship to @ cause, so It makes sense to describe Anderson as 2 malot, It also makes sense to describe his outlook as exclusively partisan: Anderson was firmly committed to a single cause, Explanation for Incorrect Answer A: Choice (A) is incorrect, A "lbertine” Is someone who is unrestrained by convention or ‘morality. To be “provincial is to have an outlook and interests that are local or restricted. Having @ local or restricted outlook might lead Anderson to support only ‘one politcal cause,” so the term “provincial” might make sense in this context. But the term “libertine” does not logically complete the sentence between having a fanatical commitment to a cause, having a provincial outlook, anc being unrestrained by morally ere Is no direct connection Explanation for Incorrect Answer C+ FromSAT Online Course Sih CUUS HA ‘SAT 2006 May Answers and Explanations Choice (C) is incorrect. A “patriot” Is someone who loves his or her country and suppor treating something with excessive lenience, generosity, or consideration. Someone who |s *fanatically committed" to a political cause might be described as a patriot, or someone who supports his or her country’s interests. However, there is no direct connection between being a patriot and maintaining an outlook that is exclusively Indulge: Its authority and interests. "Indulgent” means characterized by indulgence, or or excessively lenient or generous. Explanation for Incorrect Answer Dt Choice (D) is incorrect. A *maverick" Is an independent individual whe does not go along with a group or party. “Altruistc” means demonstrating an unselfish devotion to the welfare of others. Ifthe “politcal cause” Anderson was committed to had to do with the welfare of others, Anderson’s commitment might demonstrate an altruistic outlook, However, itis somewhat illagical to suggest that someone who Is “fanatically committed” to 2 poltcal cause is a maverick, or someone who does not go along group. Explanation for Incorrect Answer E 1 Choice (E) is incorrect. A "rebel is someone who opposes or takes arms against a government ar ruler. “Vicarious” means experienced through Imaginative or sympathetic participation in the experience of another. Although it is possible that the “political cause” Anderson was “fenatically committed to" Involved taking arms against the government, it does not make sense to say that Anderson had a "vicarous outlook, Passage 1 Many scientists tend to split the universe into two realms: science and irrationality. They believe that “truth” is only what can be experimentally proved and 4ine that “understanding” is only what fits into the straitjacket 5 — of logic. This approach may work beautifully within the scope of questions that science can answer, but life is a far richer tapestry than the threads of logic alone can weave. It Is myopic to define “reason” so narrowly that one can’t see the other threads, Science can teach us many things, 10 but there are some truths science cannot measure Passage 2 FromSAT Online Course CUUS WABI ‘SAT 2006 May Answers and Explanations After spending years training in Buddhist meditative practices, neurologist James Austin got his first taste of spiritual enlightenment while waiting on a London train platform, idly glancing toward the Thames River. Instantly, 15 the panorama of sky and buildings acquired a sense of what he calls “absolute reality, intrinsic rightness, and ultimate perfection.” He suddenly shed his formerly unshakable assumption that he was an individual, separated from the rest of the world by a skin suit. Within seconds, other 20 insights dawned. ‘Austin described in a book how this experience spurred him to study brain processes that underlie spiritual experiences, But the harsh reality of science is that those who study mysticism are rarely taken seriously. For 25 scientists, claims of mystical enlightenment have long smacked of self-deception, charlatanism, or both. ‘The “scientists” referred to in line 1, Passage 1, would most likely view the “book” mentioned in line 21, Passage 2, with io respect © curiosity c objectivity (©) ambivalence ©) skepticism ANSWERS AND EXPLANATIONS. Explanation for Correct Answer E CChoice (E) Is correct. The author of Passage 1 tells us that “many scientsts.beleve that “truth’ is only what can be experimentally proved.” These scientists would tkely view James Austin’s book with skepticism, oran attitude of doubt, because it deals with ‘spiritual experiences,” which they would kely argue cannot be proven through experiments. The author of Passage 2 even notes that scientists characterize “claims of rystical enlightenment” as “self-deception, charlatanism, or both.” Explanation for Incorrect Answer A: Choice (A) Is incorrect. The author or Passage 1 tells us that “many scientists... beleve that “ruth’is only what can be experimentally proved.” It is unlikely hat these sclentists would view James Austin’s book with respect since the book deals with “spiritual experiences,” which the scientists would Ikely argue cannot be proven through experiments, FromSAT Online Course Sih CUUS AB ‘SAT 2006 May Answers and Explanations Explanation for Incorrect Answer CChoice (B) is incorrect: The author of Passage 1 tells us that “many scientists .believe that “truth’ is only what can be experimentally proved” and states that these scientists “cantt see" things that aren’t based on logic. Therefore, itis unlikely that the scientists would view James Austin’s book with curiasity since the book deals with "spiritual experiences,” which the scientists would likely argue cannot be explained through logic or proven through experiments. Explanation for Incorrect Answer C Choice (€) is incorrect. The author of Passage 1 tells us that “many scentists..believe that “ruth’ is only what can be experimentally proved.” It is unlikely hat these scientists would view James Austin’s book with objectivity — that is, view it without any distortion from personal feelings or prejudices — because 1 1 book deals with "spiritual experiences." The scientists would likely argue that spiritual experiences cannot be explained through logle or proven way they view the book. ough experiments; this prejudice would affect Explanation for Incorrect Answer D Choice (D) is incorrect. “Ambivalence” is a simutaneous and conflicting attitude oF feeling toward something. Nothing in Passage 1 suggests that the scientists would have conflicting atutudes toward the type of experiences described in James Austin’s book, Because they have @ narrow view of *reason” and “believe that ‘ruth’ is only what can be experimentally proved,” the scient'sts would likely only view the book with skepticism, or doubt, James Austin (in Passage 2) would most likely respond to the assertion in lines 6-7 of Passage 1 ("life is. .. weave") by W/) agresing and citing his own experiance as evidence (©) agreeing and citing specific scientific findings as proof (©) dectining to offer an opinion either way (©) disagreeing and citing the beliefs of most scientists © disagreeing and citing Budchist texts for support ANSWERS AND EXPLANATIONS Explanation for Correct Answer A : Choice (A) is correct. James Austin would probably agree that “life is @ far richer tapestry than the threads of logic alone can weave"—that is, that there are meaningful and significant parts of life that don't completely fall "within the scope of que: From SAT Online Course Skt CUUS WAHL ‘SAT 2006 May Answers and Explanations science can answer"—and cite his own "taste of spiritual enlightenment" as evidence. Although his experience cannat necessarily be explained through logic, it gave him a sense of "absolute reality, intrinsic rightness, and ultimate perfection” and stimulated other “insights” Explanation for Incorrect Answer B Choice (B) is incorrect. James Austin would probably agree that “Ife is a far richer tapestry than the threads of logic alone can weave"—that Is, tha and significant parts of ife that don't completely fall"within the scope of questions science can answer"—but It is Magical to suggest that he would cite "specific scientific findings as proof.” Scientife findings would likely be defined as threads of logic.” Instead, Austin would probably cite his own “taste of spiritual enlightenment” as there are meaningful Explanation for Incorrect Answer C+ Choice (C) is incorrect: James Austin was greatly Influenced by his own “taste of spiritual enlightenment*—it gave him a sense of “absolute reality, intrinsic rightness, and ultimate perfection” ang stimulated other "insights"—so its unlikely that he would rot “offer an opinion either way.” Instead, Austin would probably agree that "Ife Is 3 far richer tapestry than the threads of logic alone can weave"—that Is, that there are meaningful and significant parts of life that don’t completely fall“within the scope of ‘questions sclence can answer." Explanation for Incorrect Answer D : CChoice (D) Is incorrect. James Austin would probably agree, not disagree, that “Iie is @ far richer tapestry than the threads of logic alone can weave"—that is, that there are ‘meaningful and significant parts of life that don’t completely fal "within the scope of questions science can answer." He Tkely would cite his own “taste of spiritual enlightenment” as evidence rather than citing “the beliefs of most scientists,” since his own experience gave him a sense of “absolute realty, intrinsic rightness, and ukimate perfection” and stimulated other “insights” Explanation for Incorrect Answer E: Choice (E) Is incorrect, James Austin would probably agree, not disagree, that “life is @ far richer tapestry than the threads of logic alone can weave"—that is, that there are ‘meaningful and significant parts of life that don’t completely fal within the scope of questions science can answer." He Tkely would cite his own “taste of spiritual enlightenment” to support his agreement, since his experience gave him a sense of “absolute realty, intrinsic rightness, anc ulimate perfection” and stimulated other “insights.” Further, "Buddhist texts” would more likely support than refute this semi- mystical view of ie FromSAT Online Course i CUUS WABI ‘SAT 2006 May Answers and Explanations ‘The author of Passage 1 would most likely characterize the “insights” mentioned in Passage 2, ine 20, as (A) mystical experiences that have litle relevance to most people's lives (©) truths that can be arrived at only through rigorous scientific studies ©) potentiatly valuable perceptions that most scientists would not consider legitimate (©) sensory impressions that are unlikely to yield meaningful information (© personal experiences that scientists should not attempt to study ANSWERS AND EXPLANATIONS. Explanation for Correct Answer C: Choice (C) is correct. The author of Passage 1 states that "Many scientists..believe that "uth" is only what can be experimentally proved," but that “there are some truths science cannot measure." The author would likely consider James Austin’s “insights” to be "potentially valuable perceptions” even If sclence cannot measure them, seeing them as part of life's “[rich} tapestry"; most scientists would likely argue that Austin’s Insights do not have a scientific explanation and are therefore not “legitimate.” Explanation for Incorrect Answer A: CChoice (A) is incorrect, There is no reason to belleve that the author of Passage 1 would argue that James Austin’s “insights” and experience *have Iitle relevance to ‘most people's lives"—he or she does not suggest that most people do not have mystical experiences.” On the contrary, the author asserts that “life isa far richer tapestry than the threads of logic alone can weave," hinting that mystical experiences ‘are not uncommon Explanation for Incorrect Answer CChoice (B) is incorrect. There is no reason to believe that the author of Passage 1 would argue thet James Austin’s “insights” are “truths that can be arrived at only through rigorous scientific studies." On the contrary, the author would likely say that ‘Austin’s experience does nat lie “within the scope of questions that science can answer” and that itis 8 "[truth] science cannot measure.” Explanation for Incorrect Answer D : Choice (D) is incorrect. Although the author of Passage 1 refers to “truths science cannot measure,” he or she does not discuss the Idea that experiences like James Austin’s (even those that do not "[fit] into the stratjacket of logic") are “unlikely to FromSAT Online Course Sih CUUS HA ‘SAT 2006 May Answers and Explanations Yield meaningful information.” Rather, the author of Passage 1 would likely consider further research Into these experiences. Explanation for Incorrect Answer E+ CChoice (E) Is incorrect. Although the author of Passage 1 states that “there are some truths science cannot measure,” he or she does not suggest tat “scientists shoulé not attempt to study” the type of personal experience that led to James Austin’s “insights.” Rather, the author of Passage 1 would likely suggest further study Unlike Passage 1, Passage 2 makes use of (8) generatiation V1 anecdote (©) figurative language (©) iterary allusion © technical jargon ANSWERS AND EXPLANATIONS Explanation for Correct Answer CChoice (B) is correct, An anecdote Is @ short narrative of an interesting, amusing, or biographical incident, The author of Passage 2 makes use of an anecdote when he or she relates a short narrative about James Austin’s “first taste of spiritual enlightenment." Passage 1 does not make use of an anecdote Explanation for Incorrect Answer A: Choice (A) is incorrect. The author of Passage 1 makes use of generaliaition, or draws general conclusions, when he or she describes the thinking of "many scientists” concerning “truth” and “understanding.” The author of Passage 2 makes use of ‘generaliation when he or she describes wha ‘mystical enlightenment.” scientists” think about “claims of Explanation for Incorrect Answer C : Choice (€) is incorrect. The author of Passage 1 makes use of figurative language; the passage contains a metaphor describing lfe as a “tapestry” with “threads of logic. Explanation for Incorrect Answer D: Choice (D) is incorrect. literary allusion is a reference to a character or event in ‘nother work of literature, usually one that is well-known. Although the author of, Passage 2 discusses a non-fiction book written by James Austin, neither passage conta FromSAT Online Course Sih CUUS HA ‘SAT 2006 May Answers and Explanations 1s @ literary allusion, Explanation for Incorrect Answer E Choice (E) Is incorrect, Nelther passage makes use of technical jargon, or the terminology of a special, usually high-trained group. The following passage by an Asien American author has been adapted from a short story published in 2000. Caroline and Marcela are former friends; Deen is Caroline's current boyfriend, Caroline Yip and Marcella Ahn had a history. They had both lived in Cambridge, Massachusetts, in their twenties, and for several years they had been the best of friends — tine inseparable, really. But then their first books had come 5 out at the same time, Marcella’s from a major New York publisher, Caroline's from a srrall, albeit respected press. Both had very similar jacket photos, the two women looking solemn and precious, hair flowing in full regalia ‘An unfortunate coincidence, Critics couldn't resist 40 reviewing them together, mocking the pair, even then, as “The Hair Poets,” “The Braids of the East,” and “The New Asian Poe-tresses,” But Marcella escaped these barbs relatively unscathed Her book was taken seriously, and she was even compared 15 to Marianne Moore and Emily Dickinson. Her poetry was highly erudite, usually beginning with mundane observations about birds or plant life, then slipping into long, abstract meditations on entropy and inertia, the Bible, evolution, and death, punctuated by the briefest mention of 20 personal deprivations. Or so the critics said. Dean still had the book from the library, and he couldn't make heads or tails of it In contrast, Caroline's book had been skewered, She wrote in a slangy, contemporary voice, full of topical, 25 popculture allusions. She wrote about Marilyn Monroe and moo goo gai pan, about alien babies and her strange, loopy obsession with poultry. She was roundly dispatched as a mediocre talent Worse, Caroline sald, was what happened afterwards, 3 Marcella began to thwart her at every turn. Teaching jobs, coveted magazine publications, awards—everything Caroline applied for, Marcella got. It didn’t hurt that FromSAT Online Course CUUS ABN ” “6 ‘SAT 2006 May Answers and Explanations Marcella was a shameless schmoozer, flirting and networking with anyone who might be of use, all the while ridiculing them behind their backs. The fact was, Marcella was rich. Her father was a shipping tycoon, and she had a trust fund in the millions. She didn’t need any of these pitifully small sinecures which would have meant 2 livelihood to Caroline, and it became obvious that the only reason Marcella was pursuing them at all was to taunt her. °She’s a vulture, a vampire," Caroline told Dean. "You know she won’t go out in the light of day? She stays up until four, five in the morning and doesn’t wake up until past noon.” ‘And then there was the matter of Evan Paviromo, the English-Italian editor of a literary journal whom Caroline had dated for seven years, waiting patiently for them to get married and have children. He broke it off one day without explanation. She dogged him. Why? Why was he ending it? She refused to let him go without some sort of answer. Finally he complied, “It’s something Marcella said,” he admitted, {At first Caroline feared they were involved romantically, but the truth was more vicious, "Marcella told me she admired me," Evan said, “that I was far more generous than she could ever be. She said she just wouldn't be able to stay with someone whose work she didn’t really respect. I thought about that, and I decided I’m not that generous after all. It’s something that would eat away at me, that’s bothered me all along. It's something I can’t abide.” Caroline fled to California, eventually landing in the little nondescript town of Rosarita Bay. She completely disengaged herself from the poetry world. She was still writing every day, excruciating as it was for her, but she had not attempted to publish anything in six years, She was thirty-seven now, and a waitress—the breakfast shift ata diner, the dinner shift at a barbecue joint. Her feet had grown a full size from standing so much, and she was broke. But she had started to feel like her old self again, healthier, more relaxed, sleeping better. Dean had a lot to do with it, she said. She was happy—or as happy as it was possible for a poet to be, Until now, Until Marcella Ahn suddenly arrived. The word “history” (line 1) refers to @ From SAT Online Course it CLUS vo 8b ‘SAT 2006 May Answers and Explanations (®) chronological record of actual occurrences ©) long-standing and noteworthy relationship (©) formal writen account of an incigent (©) branch of knowledge focused on the past (©) creative work based on historical events ANSWERS AND EXPLANATIONS Explanation for Correct Answar| Choice (B) is correct. In the first sentence the author introduces the topic of the passage: the “long-standing and noteworthy relationship” between Caroline and Marcella. The poets were once "the best of friends” and even published their first books. at the same time, but over the years "Marcella began to thwart [Caroline] at every turn," and Caroline eventually “disengaged herself from the poetry world.” The poets’ “history” Is thelr relationship. Explanation for Incorrect Answer A: Choice (A) is incorrect. Although the passage does chronologically recount actual ceccurrences in the relationship between Caroline and Marcella ("both lived in Cambridge..in thelr twenties,” “everything Caroline applied for, Marcella got,” "Caroline fled to California”), the author does not suggest that the women possessed any sort of record of these occurrences. Rather, the author uses the word “history” to introduce the idea that the poets’ relationship is “long-standing and noteworthy.” Explanation for Incorrect Answer C1 Choice (C) is incorrect: The authar uses the ward "history" to intracuce the idea that the relationship between Caroline and Marcella is “long-standing and noteworthy,” not to suggest that the women had a “formal written account” of any one of the incidents later described in the passage Explanation for Incorrect Answer D : Choice (b) is incorrect. The word *history” refers to the “long-standing ané noteworthy relationship” between Caroline and Marcella, not to the subject of history as the “branch of knowledge focused on the past.” Rather, this sense of history would be used to describe "the history department” or “a history class.” Explanation for Incorrect Answer E: Choice (E) is incorrect, The author uses the word “history” to introduce the events in Caroline end Mercella’s “long-standing and noteworthy relationship," not to suggest that the women had 2 “creative wark based on historical events.” FromSAT Online Course CUUS WABI ‘SAT 2006 May Answers and Explanations In context, what is the primary significance of the two similar “jacket photos’ dine 792 9) They served as a way for erities to link two newly published poets. (©) They provided an opportunity for two poets to announce their Friendship (©) They revealed the animosity that had always existed between the two poets. (©) They mirrored the similarity of topics in the two books of postry. (© They reflected the casualness and informality of the language used in the poems. ANSWERS AND EXPLANATIONS: Explanation for Correct Answer A: CChoice (A) is correct. The similar jacket photos gave critics a way to “ink two newly published poets.” Each woman appeared In her photo “looking solemn” and displaying “hal flowing in full regal Caroline Yip and Marcella Ahn as.a pair and gave them nicknames such as "The Hair Poets,’ The Braids ofthe East,’ and "The New Asian Poe-tresses. +; because of this “unfortunate coincidence," erties reviewed Explanation for Incorrect Answer B Choice (B) is incorrect, The passage suggests that Caroline Yip and Marcella Ahn were friends when their first books were published, but there is no indication that the similar Jacket photos allowed the women to "announce thelr friendship,” Rather, the author refers to the similarity of the photos as “an unfortunate coincidence” because erties linked the twa poets, “mocking the palr.” Explanation for Incorrect Answer C CChoice (C) is incorrect: Although the passage suggests that animosity existed between Caroline Yip and Marcella Ahn at some point in their relationship, animosity did not always exist between the two poets. On the contrary, the women were once best of friends. Explanation for Incorrect Answer D : Choice (D) is incorrect. The similarity of the jacket photos did net mirror the similarity of the topics in the poets’ books. The passage tells us that the topics in the books were. very different: Marcella’s poetry was “highly erudite,” or profound, including both “mundane observations” and “abstract meditations” on a variety of subjects, while Caroline's poetry was “slangy,” "contemporary," and “full of..opculture allusions. Explanation for Incorrect Answer FromSAT Online Course Sith CUUS WAIN ‘SAT 2006 May Answers and Explanations Choice (E) Is incorrect. The similar jacket photos did not reflect “the casualness and Informality of the language used” in Caroline Yip's ané Marcella Ahn’s poetry; the passage suggests that the jacket photos were not casual and informal. Rather, the women are described as appearing "solemn and precious,” or serious ané refined, with thelr “hair flowing in full regalia,” or appearing to be royal. Additionally, the passage states that Yip's poetry contains informal language ("slangy, contemporary voice”) but oes not address the formalty of Ann's language. ‘The tone of the characteriations quoted in lines 11-12 is best described as () morose © curious WS) sardonic o threatening © incredulous ANSWERS AND EXPLANATIONS: Explanation for Correct Answer C Choice (C) is correct. “Sardonic” means disdainfully humerous er derisively mocking, ‘The passage states that due to the “unfortunate coincidence” of both Caroline Vip and Marcella Ahn appearing in their jacket photos with “hair lowing in full regalia,” erties ‘mocked the pair by calling them "The Hair Poets,’ ‘The Braids of the East,’ and "The New Asian Poe-tresses."" These mocking characteriations are sardonic in tone. Explanation for Incorrect Answer A: Choice (A) is incorrect. *Morase” means sullen and gloomy. The tone of the characteriztions ("The Hair Poets,” "The Braids of the East,” "The New Asian Poe- tresses") is derisively mocking, not sullen and gloomy. Explanation for Incorrect Answer B Choice (6) is incorrect, “Curious” means marked by desire to Investigate. The tone of the characterimtions ("The Hair Poets,” "The Braids of the East," "The New Asian Poe- tresses") Is derisively mocking; the characteriations do not reveal the critics’ desire to Investigate the poets. Explanation for Incorrect Answer Dt Choice (D) is incorrect. “Threatening” means menacing or showing intent to cause ugh the characteriastions (“The Hair Poets,” *The Bralds of the East," "The New Asian Poe-tresses”) are derisively mocking and could be harmful to the posts’ harm. Al FromSAT Online Course Ski CUUS WSK ‘SAT 2006 May Answers and Explanations book sales and self-esteem, they do nat contain direct threats or reveal plans to cause harm, Explanation for Incorrect Answer E: Choice (E) Is incorrect. “Incredulous” means disbelieving or unable to accent something as true. The tone of the characteriaations (“The Hair Poets," “The Braids of the East," “The New Asian Poe-tresses") Is derisively mocking; the characteriztions do not suggest that t 1 critics were unable to accep! the poets or their work as true ‘The tone of the statement in line 20 ("Or. .. said’) is best described as () impatient © apologetic (©) reflective ©) anxious unconvinced ANSWERS AND EXPLANATIONS: Explanation for Correct Answer E : Choice (F) Is correct, The narrator of the passage describes Marcelle’s poetry as Incorporating "abstract meditations on entropy and inertia,” among other things. The narrator then notes, “Or so the critics sald,” adding that Caroline's boyfriend, Dean, “had the book from the library” and “couldn't make heads or tals of i.” The tone of the statement "Or so the criti sald” can be described as unconvinced. The narrator clearly sympathizes with Caroline, not Marcella, and uses subtle hints such as this phrase to suggest that while Marcella may have pleased the erities, she has not convinced everyone of her superiority Explanation for Incorrect Answer A: Choice (A) is incorrect. The tone of the statement "Or so the critics said” is best, described as unconvinced, not impatient, The narrator of the passage describes Marcella's poetry and then notes, “Or so the ertics said," before adding that another character, Dean, “couldn't make heads or tails of Marcella’s poetry. The narrator clearly sympathizes with Caroline, not Marcella, and uses subtle hints such as this phrase to suggest that while Marcella may have pleased the critics, she has not convinced everyone of her superiority. Nothing suggests that the narrator is impatient, fr anxious for something to happen, Explanation for Incorrect Answer From SAT Online Co urse i CLUS ABI ‘SAT 2006 May Answers and Explanations Choice (B) is incorrect. The narrator af the passage describes images found in Marcella's poetry and then notes, “Or so the critics said," before adding that another character, Dean, “couldn't make heads or tails of” Marcella's poetry. Nothing suggests that the narrator is offering an apology to anyone. The tone of the statement critics said” is better described as unconvinced; the narrator suggests that not ‘Or so everyone who reads Marcella’s poetry is convinced of her superlarity Explanation fer Incorrect Answer C : Choice (C) is incorrect, The narrator of the passage describes Marcella’s poetry and then notes, “Or s0 the erities said,” before adging that another character, Dean, “couldn't make heads or tails of” Marcella’s poetry. The tone of the statement "Or s0 the crities said” is not yest described as reflective, or thoughtful and deliberatve. Rather, the tone Is unconvinced; the narrator suggests that not everyone who reads Marcella's poetry is convinced of her superiority. Explanation for Incorrect Answer D: Choice (D) is incorrect. The narrator of the passage describes Marcella's poetry and then notes, "Or so the critics sald,” before adcing that another character, Dean, “couldn't make heads or tals of” Marcella’s postry. Nothing suggests that the narrator Is anxious. The tone of the statement “Or so crites said” Is better described as, Lunconvinced; the narrator suggests that not everyone who reads Marcella's aoetry is convinced of her superiority, Lines 25-27 ("She . . . poultry") serve to provide examples of WA) particular references found in Caroline's poetry (©) diverse subjects about which Caroline knew surprisingly lithe (©) allusions in Caroline's poetry that puzded critics (©) topies that should not be mentioned even in modern postry (© images that are found also in Marcelle’s poetry ANSWERS AND EXPLANATIONS: Explanation for Correct Answar A: CChoice (A) is correct. In the third paragraph the passage describes Caroline's poetry, Wich critics called “mediocre.” Lines 25-27 provide examples of the types of things Caroline wrate about by listing particular references found In her poems: "She wrote about Marilyn Monroe and meo goo gai pan, about alien babies and her..obsession with pouty.” Explanation for Incorrect Answer FromSAT Online Course Siti CUUS WABI ‘SAT 2006 May Answers and Explanations Choice (B) is incorrect. Although the subjects listed in lines 25-27 are diverse, there Is ro indication that Caroline knew “surprisingly litle” about them, On the contrary, Caroline would have known her awn “strange, loopy obsession with poultry” very well Explanation for Incorrect Answer C Choice (C) is incorrect. Cries calles Caroline “a mediocre talent," but nothing in the passage suggests that they were “puzzled” by the “topical, popculture allusions” in her poetry. The subjects listed in fines 25-27 are examples of particular references found in Caroline's poems, Explanation for Incorrect Answer D Choice (D) is incorrect. While the narrator implies that Caroline's poetry may have been less successful because it dealt with somewhat unorthodox topics, nothing In the passage suggests that critics thought the subjects sted in lines 25-27 are “topics that should not be mentioned even in modern poetry." Rather, lines 25-27 provide examples of particular references found in Caroline's poems. Explanation for Incorrect Answer E CChoice (E) Is incorrect. The passage does not suggest that the topics listed in tines 25- 27 are also found in Marcella’s poetry. We are told that Marcella wrote about “birds or plant life” and "entropy and inertia, the Bible, evolution... nothing suggests that she also wrote about “Marilyn Monroe and moo goo gai pan." Lines 25-27 simply provide examples of particular references found in Caroline's poems. ‘The “reason” mentioned in line 40 was most “obvious” (ine 39) to (®) magaane publishers © award committees () marcella's father ©) dean ©) caroline ANSWERS AND EXPLANATIONS: Explanation for Correct Answer E : CChoice (E) is correct. The fourth paragraph presents Caroline's view of her relationship with Marcelle afer their frst books were published (*Worse, Caroline saié, was what happened afterwards"). While itis “obvious” to Caroline that Marcella only pursued “teaching jobs, coveted magedne publications, [and] awards” — things Marcella “didn’t need" — in order to “taunt” Caroline, this view is likely biaseé. Only Caroline could From SAT Online Co urse i CLUS ABI ‘SAT 2006 May Answers and Explanations believe that Marcella's main motivation to succeed as a poet was the desire to taunt Caroline Explanation for Incorrect Answer A: CChoice (A) is incorrect. The fourth paragraph presents Caroline's view of her relationship with Marcella after their first books were published — “its obvious" to Caroline that Marcella only pursued certaln “coveted magazine publications” In order to taunt” Caroline. The magane publishers’ thoughts concerning Marcella’s decisions are not addressed, Explanation for Incorrect Answer Choice (B) is incorrect. The fourth paragraph presents Caroline's view of her relationship with Marcella after thelr first books were published — Its “obvious” to Caroline that Marcella only pursued certain “awards” in order to “taunt” Caroline. The award committees’ thoughts concerning Marcella's decisions are not addressed Explanation fer Incorrect Answer C: Choice (C) is incorrect, The fourth paragraph presents Caroline's view of her relationship with Marcella after their first books were published. Caroline does mention Marcellas father when describing Marcella’s wealthy background, but there is no suggestion that he thinks the “reason” for any of Marcella's behavior is “obvious.” Its Caroline herself who finds the reason to be obvious; she feels that Marcella only pursued certain things in order to “taunt* Caroline. Explanation for Incorrect Answer D : Choice (D) is incorrect. Although Caroline tells Dean that she thinks Marcella is "a vulture, a vampire,” there is no suggestion that Dean belleves the “reason” for any of, Marcella's behavior to be “obvious.” It is Caroline herself who finds the reason to be obvious; she feels that Marcella only pursued certain things In order to “taunt” Caroline In context, ines 45-50 ("And . . . answer") suggest that Caroline’s response to Evan's action encompassed all of the following EXCEPT () shock © aisbetier (©) contusion ©) persistence WE) retaliation ANSWERS AND EXPLANATIONS: From SAT Online Co ese kth CU ‘SAT 2006 May Answers and Explanations Explanation for Correct Anwar E Choice (E) is correct. Caroline's response to Evan's action—breaking off their relationship suddenly and “without explanation*—Includes shock, disbelie, confusion, and persistence, She does not respond by retaliating, or fighting back, against Evan or Marcella; Caroline merely demands "some sort af answer" that will explain Evan's ecision, Explanation for Incorrect Answer A: Choice (A) is incorrect. The narrator states that Caroline was "waiting patiently” to marry Evan when Evan suddenly broke off the relationship “without explanat Caroline's response—dogging Evan to explain “why” and not *[etting] him go without some sort of answer" reveals her shock at his decision. Explanation for Incorrect Answer B Choice (B) is incorrect, The narrator states that Caroline was *walting patiently” to marry Evan when Evan sudéenly broke off the relationship "without explanation. Caroline's response—"[refusing] to let him go without” explaining why he was “ending it"—reveals her disbelief. Explanation for Incorrect Answer C Choice (C) is incorrect. The narrator states that Caroline was “waiting patiently” to marry Evan when Evan sudcenly broke off the relationship "without explanation.” Caroline's response—{refusing] to let him go without” explaining why he was “ending It"—reveals her contusion, Explanation for Incorrect Answer D Choice (D) is incorrect. When Evan suddenly broke off his relationship with Caroline “without explanation,” Caroline responded by *[refusing] to let him go without” explaining why he was “ending it.” This response shows Caroline's persistence. Which is most analogous to the situation described in lines 62-70 ("She better")? (A) A ballerina achieves early success and fame but then is forced by personal problems to stop dancing, (© A novelist, undaunted by many inital rejections, continues to try to publish her stories VC) An artist is unable to support himself with his painting but finds contentment working as 2 clerk FromSAT Online Course Sih CUUS HAH ‘SAT 2006 May Answers and Explanations (©) A scholar, discouraged by continuous harsh criticism of his writings, gradually becomes despondent. © A viotnist, skilled at interpreting the works of others, is unsuccessful at composing her own music ANSWERS AND EXPLANATIONS. Explanation for Correct Answer C: Choice (C) is correct. In lines 62-70 we learn that in California Caroline was writing but ‘ot publishing anything, and working as @ waitress, yet she was also starting to feel “nealthier" and "mare relaxed.” An artist being unable to support himself with his paintings but finding contentment working as. clerk is most analogous to this situation in both situations someone is not financially supported by his or her creative work but finds contentment, or satisfaction, in an unrelated job. Explanation for Incorrect Answer A : Choice (A) is incorrect, A ballerina achieving early success and fame and being forces by personal problems te stop dancing is not most analogous to the situation described In lines 62-70, In those lines we learn that in Calfornia Caroline was writing but nat publishing anything, and working as a waitress, yet she was also starting to feel “healthier” and “more relaxed.” Because Caroline was not supporting herself with her creative work but found a level of contentment in an unrelated job, the most analogous situation is the painter who finds contentment working as @ clerk. Explanation for Incorrect Answer B Choice (6) is incorrect, A novelist who continues to try to publish her stories despite Initial rejections is nat most analogous to the situation described in lines 62-70. On the contrary, in line 65 we learn that Caroline *had not attempted to publish anything in sb years.” Because Caroline was not supporting herself with her creative work but found a level of contentment in an unrelated job, the mast analogous situation isthe painter who finds contentment working as a clerk, Explanation for Incorrect Answer D: Choice (D) is incorrect. A scholar who is discouraged by harsh criticism of his writings and begins to feel despondent is not the mast analogous to the situation described in lines 62-70. Although Caroline may have been despondent at one time because of the crticism of her poetry, in lines 62-70 we learn that after moving to Califernia she began working as a waltress and “started to feel lke her old self again,” feeling healthier" and *more relaxed.” Because Caroline was not supporting herself with her creative work but found a level of contentment in an unrelated job, the most analogous situation isthe painter who finds contentment working as 2 clerk. FromSAT Online Course CUUS WABI ‘SAT 2006 May Answers and Explanations Explanation for Incorrect Answer E+ Choice (F) is incorrect. A violinist who is skilled at interpreting the works of others but, unsuccessful escribed in lines 62-70, In those lines we learn that after Caroline moved to California she continued to write, though she did not attempt to publish anything. She also began working as a waitress and "started to feel like her old sel again,” feeling “healthier” and "more relaxed.” Because Caroline was not supporting herself with her creative Work but found a level of contentment in an unrelated job, the most analogous situation fs the painter who finds contentment working as 2 clerk. composing her own music is nat the most analogous to the situation ‘The statement in lines 71-72 ("She .. . be") suggests which ofthe following about poets? (8) They tend to be reclusive and uncommunicative 18) They are by nature inclined to be discontented. (©) They rarely reveal their true feelings. (©) Their poetry is shaped by real and imaginary experiences, (© Those who are most successful express a range of emotions in their poems. ANSWERS AND EXPLANATIONS: Explanation for Correct Answar Choice (B) is correct. The somewhat humorous remark that Caroline was "as happy as It was possible for a poet to be” suggests that posts are not usually happy. In other words, the comment implies that poets “are by nature Inclined to be discontented,” or dissatisfied Explanation for Incorrect Answer A: Choice (A) is incorrect, Although Caroline “disengaged herself fram the poetry world,” nothing in the passage suggests that she was reclusive or uncemmunicative in general Instead, the remark that Caroline was "as happy as it was possible fora poet at poets are nat naturally very happy, tending bet somewhat humorously suggests Instead "to be discontented.” Explanation for Incorrect Answer C+ Choice (C) is incorrect, The remark that Caroline was “as happy as it was possible for a poet to be” does not suggest thet posts “rarely reveal thelr true feelings"; Caroline's feeling of happiness does seem to be true, and there is no reason to belleve she did not reveal it. Instead, the somewhat humorous remark suggests that instead of being happy, poets "are by nature inclined to be discontented, urse i CLUS ABI From SAT Online Co ‘SAT 2006 May Answers and Explanations Explanation for Incorrect Answer D: Choice (D) is incorrect. The remark that Caroline was "as happy as it was possible for a poet to be” dows not address the idea that a poet's work "is shaped by real and Imaginary experiences.” Rather, the somewhat humorous remark suggests that poets are nat naturally very happy, tending instead “to be discontented.” Explanation for Incorrect Answer E: Choice (E) Is incorrect, The remark that Caroline was “as happy as it was possible for a poet to be” does nat suggest that posts "who are mast successful express @ range of emotions in their poems"; the passage mentions Caroline's emotions ("She was happy") but does not address the idea that her emotions should be expressed in her poetry Instead, the somewhat humorous remark suggests that poets are not naturally very happy, tencing instead “to be discontented.” This passage is excerpted from a 1996 book about Thomas Jefferson, statesman and third President of the United States. According to the National Park Service, about a million tourists pay their respects to Thomas Jefferson in his memorial each year. On the March day in 1993 that I Line visited, several hundred tourists walked up the marble 5 steps and looked up to the four inscribed panels on the walls and read the words, often moving their lips and murmuring the famous phrases to themselves, The first panel, which attracted more attention than the others, contained the most farrous and familiar words in American 10 history: “We hold these truths to be self-evident, that all men are created equal, that they are endowed by their Creator with certain inalienable Rights, that among these are Life, Liberty, and the pursuit of Happiness.” But what do these words mean? How do they create 45 magic? Merely to ask these questions is to risk being accused of some combination of treason and sacrilege, since sel evident truths are not meant to be analyzed: that is what being self-evident is all about. But when these words are 20 stripped of the patriotic haze, read straightaway and liter- ally, two monumental claims are being made here. The explicit claim is that the individual is the sovereign unit in society; the individual's natural state is freedom from and equality with all other individuals; this is the natural FromSAT Online Course CUUS WABI ‘SAT 2006 May Answers and Explanations 25 order of things. The implicit claim is that all restrictions on this order are immoral transgressions, violations of what God intended; individuals liberated from such restrictions will interact with their fellows in a harmonious scheme requiring no extemal discipline and producing maximum 30 happiness. This is a wildly idealistic message, the kind of good news simply too good to be true. It is, truth be told, a rec- ipe for anarchy. Any national government that seriously attempted to operate in accord with these principles would 35 be committing suicide, But, of course, the words were not intended to serve as an operational political blueprint. Jefferson was not a profound political thinker. He was, however, an utterly brilliant rhetorician and visionary The genius of his vision is to propose that our deepest 40 yearnings for freedom are in fact attainable. The genius of his rhetoric is to articulate irreconcilable human urges at a sufficiently abstract level to mask their mutual exclusiveness. Jefferson guards the American Creed at this inspirational level, which is inherently immune to 45 scholarly skepticism and a place where ordinary Americans can congregate to speak the magic words together. The Jeffersonian magic works because we permit it to function ata rarified region where real-life choices do not have to be made, 50 Jefferson's words allow American citiaens to come together and simultaneously embrace seemingly opposite propositions. They can believe, for example, that health care and a clean environment for all Americans are natural rights, but that the federal bureaucracies and taxes required 55 to implement medical and environmental programs violate individual independence. The primal source of Jefferson’s modern-day appeal is that he provides the sacred space— not really common ground but more a midair location floating above all the political battle lines —where 40 Americans can come together on such issues and, at least for that moment, become a chorus instead of a cacophony. Lines 3-7 ("On the... themselves") primarily serve to (8) emphasize the austere beauty of the Jefferson Memorial (© suggest that many Americans are unfamiliar with Jefferson's philosophy ©) indicate the reverence people feel for Jefferson’s words From SAT Online Course At CLUS v8 ‘SAT 2006 May Answers and Explanations (©) show the strong regard that people have for history (© point out that Americans experience different degrees of patriotism ANSWERS AND EXPLANATIONS: Explanation for Correct Answer C: Choice (C) is correct. Reverence is honor or respect that is felt or shown. The author describes “several hundred! visitors to the Jefferson Memorial reading the words Inscribed on the monument. The fact that the tourists are stopping to read Jefferson's well-known words, often “murmuring the famous phrases to themselves," shows the respect they have for Jefferson's words an¢ ideas. Explanation for Incorrect Answer At Cchoice (A) is incorrect. Although the Jefferson Memorial may be described as an ‘austere, or plain and formal, bullding, ines 3-7 focus mainly on the tourists" reaction to seeing Jefferson's words inscribed on the monument’s panels. The primary point of these lines is to display the public's respect for Jefferson's famous words, not describe the monuments overall appearance. Explanation for Incorrect Answer Choice (8) is incorrect. The author describes the tourists "murmuring the famous phrases" Inscribed on the Jefferson Memorial to themselves, The author is not suggesting that the tourists are unfamiliar with Jefferson’s ideas. Rather, by specifying that the phrases are “famous,” the author implies that most people are familar with Jefferson's philosophy. Explanation for Incorrect Answer D Choice (D) is incorrect. Although the visitors to the monument may have a strong regard or respect for history In general, the author focuses specifically on the Jefferson Memorial. The primary point of the sentence in lines 3-7 isto éisplay the public's respect for Jefferson's famous words. Explanation for Incorrect Answer E+ Choice (E) is incorrect. By describing how “several hundred tourists” recite the words inscribed on the panel, the author focuses on the respect the public shows for Jefferson's words. There is no indication in these lines that some Americans are more. patriotic than others—although it is certainly the case ‘The author suggests that "to ask these questions” (line 16) Is to risk being considered FromSAT Online Course Sih CUUS AB ‘SAT 2006 May Answers and Explanations () ignorant ©) insincere WAS) heretical (©) sentimental © impractical ANSWERS AND EXPLANATIONS: Explanation for Correct Answer C Choice (C) is correct. The author states that "merely to ask these questions Isto risk being accused of some combination of treason and sacrilege.” The correct answer must be similar in meaning to “treason"—the betrayal of one's country—and "sacrilege'—a lack of reverence for a sacred object or idea. The term “heretical,” which means characteriasd by departure from accepted beliefs or standards, isthe best ft. Explanation for Incorrect Answer A : CChoice (A) is incorrect. The author states that "merely to ask these questions Is to risk being accused of some combination of treason and sacrilege.” The correct answer must be similar in meaning to “treason"—the betrayal of one's country—and "sacrileg lack of reverence for a sacred object or idea. The term “ignorant” means lacking knowledge; being ignorant is very different from commiting treason or sacrilege, Explanation for Incorrect Answer B Choice (B) is incorrect, The author states that "merely to ask these questions Is to risk being accused of some combination of treason and sacrilege.” The correct answer must be similar in meaning to “teason"—the betrayal of one's country—and "sacrile lack of reverence for a sacred object or idea. The term “insincere” means false or hypocritical; being insincere is nat similar to commit 19 treason o sacrilege. Further, the author does seem to be sincere when asking these questions, Explanation for Incorrect Answer Dt Choice (D) is incorrect. The author states that "merely to ask these questions isto risk being accused of some combination of treason and sacrilege.” The correct answer must be similar in meaning to “treason"—the betrayal af one's country—and “sacrileg lack of reverence for a sacred object or dea. The term “sentimental” means resulting from emotion rather than reason or thought; being sentimental is not similar to committing treason o sacrilege. Further, by questioning how Jefferson's ideas “create magic," the author snows reason and thought, not emotion. Explanation for Incorrect Answer E 1 FromSAT Online Course i CUUS ASIN ‘SAT 2006 May Answers and Explanations Choice (E) is incorrect. The author states that "meraly to ask these questions is to risk being accused of some combination of treason and sacrilege.” The correct answer must be similar in meaning to “treasan"—the betrayal of one's country—and "sacrilege"—a lack of reverence for a sacred object or idea. The term “impractical” means not sensible, being impractical is very different from committing treason or sacrilege. ‘The passage as a whole suggests that the "two monumental claims” (line 21) are (8) evidence of Jefferson's profound political thinking (© useful principles that can help to solve everyday problems (©) more important to Americans than they probably should V1) incompatible when put into practice (© irrelevant at this point in American history ANSWERS AND EXPLANATIONS Explanation for Correct Answar D : Choice (D) is correct. The author explains in the fourth paragraph that a “national government that seriously attempted to operat bbe committing suicide." The author makes it very clear that—when “read straightaway in accord with these principles woule and tterally’—what Jefferson proposes is an ideal that would never work in real-life situations. Explanation for Incorrect Answer A Choice (A) is incorrect. In the fourth paragraph the author states that Jefferson "was rot @ profound politcal thinker." Moreover, the author explains earlier in that paragraph that a “national government that seriously attempted to operate in accord with these principles would be committing suicide.” The author makes it very clear that—when "read straightaway and Iiterally”—what Jefferson proposes Is an ideal that Would never work in real-life situations Explanation for Incorrect Answer Choice (B) is incorrect, The author explains in the fourth paragraph that a “national government that seriously attempted to operate in accord with these principles woule be committing suicide." The author makes it very clear that—when "read straightaway and lterally’—what Jefferson proposes is an ideal that would never work in real-life situations, not an ideal that would be useful in solving everyday problems, Explanation for Incorrect Answer C: FromSAT Online Course Sih CUUS HA ‘SAT 2006 May Answers and Explanations Choice (C) is incorrect. Despite the fact that these claims put forth an ideal that would not work in real-ife situations (2 "national government that seriously attempted to operate in accord with these principles would be committing suicide"), the author does rot imply that they are more important to Americans than they should be. In fact, the author shows that the claims are of crucial importance: "Jefferson's words allow American chtaens to come together.” Explanation fer Incorrect Answer E CChoice (E) Is incorrect, Although the author questions whether Jefferson's ideas canbe ‘applied to everyday situations, the author states in line 47 that the “Jeffersonian magic works.” Jefferson's claims are by no means Irrelevant; rather, “Jefferson's words allow American citizens t come togethe ‘The author's view of “the implicit claim" (line 25) might best be characterized as (8) dismissive © defensive (©) revere (©) resentful WE) skeptical ANSWERS AND EXPLANATIONS Explanation for Correct Answer E = Choice (E) Is correct. The Implict, or implied, claim, as interpreted by the author, is that all restrictions on human freedam are bad. As the author explains, “individuals liberated from such restrictions will interact with their fellows in a harmonious scheme requiring no external discipline.” As the rest of the passage makes clear, the author is skeptical, or full of doubt, as to the truth of this claim, An example of the author's skepticism can be found in lines 31-32: "This is 2 wildly idealistic message, the kind of 00d news simply too good to be true. Explanation for Incorrect Answer A: Choice (A) Is incorrect. To be dismissive of something Is to refuse to even consider that something might be true. Although the author questions Jefferson's claim about equality and individual freedom, the author clearly gives serious consideration to these Ideas, The author is not dismissive of the claim, Explanation for Incorrect Answer CChoice (B) is incorrect. To be defensive is to constantly protect oneself fram criticism, FromSAT Online Course kh CUUS HERS, ‘SAT 2006 May Answers and Explanations AAltiough the author defends, or supports, the argument with éetalls and reasoned opinions, the author is not being defensive, Rather, the author clearly acknowledges the risk one takes by asking questions about Jefferson's words ("Merely to ask these questions Is to risk being accused of some combination of treason and sacrilege"). The author does not seem to mind being criticize Explanation for Incorrect Answer C: Choice (C) is incorrect. To be reverent is to express awe or respect. Alt ‘author may feel an overall respect for Jefferson as a writer and visionary, the author is far from being in awe of this particular claim. As the rest of the passage makes cleer, the author is skeptical, or full of doubt, as to the truth of this claim, ough the Explanation for Incorrect Answer D: Choice (D) is incorrect. To be resentful is to feel annoyance or ill wil. Akhough the ‘author may disagree with Jefferson's implict claim, nothing in the passage suggests that the author feels resentful ‘The author uses the phrase “rarified region” (Ine 48) to hint that only a few people truly appreciate the originality of Jefferson’s philosophy indicate that Jefferson accurately predicted the political problems of the future © suggest that Jefferson’s world was remote from the realities faced by Americans today o imaly that Jefferson's ideas are too obscure for the average person to grasp ©) emphasian the unique status that Jefferson's assertions enjoy ANSWERS AND EXPLANATIONS: Explanation for Correct Answer E = Choice (F) is correct. ‘Rarfied” is the opposite of "down to earth.” Something that is rarified is lofty, philosophical, spiritual. The phrase "raried region" perfectly describes that "inspirational level” the author refers to, where “ordinary Americans can congregate to speek the magic words together." The phrase "rarined region” cemphasias the fact that Jefferson's "magic words" occupy a unique status “where real- lite choices do not have to be made, Explanation for Incorrect Answer A: Choice (A) Is incorrect, Although the author describes Jefferson’s ideas as occupying rarfied” of lofty region, the author states this region provides a” place where ordinary FromSAT Online Course kh CUUS HERS, ‘SAT 2006 May Answers and Explanations Americans” can come together. The author does not suggest that only a few people appreciate Jefferson's ideas. Explanation for Incorrect Answer Choice (B) is incorrect. Although the author describes Jefferson as a visionary, oF someone who has great or unusual foresight and imagination, he or she does not suggest that Jefferson actually prealcteatuture political problems. Rather, the author Uses the phrase “rarefied region” to emphasia the fact that Jefferson's "magic words” occupy 2 unique status "where realslfe choices do not have to be made.” Explanation for Incorrect Answer C Choice (C) is incorrect. Although the author claims that Jefferson's ideas "function at @ rarified region,” itis not because Jefferson's world was remote from modern realities. ‘The author explains that Jefferson's ideas are still relevant today, despite the fact that he lived two hundred years ago; he was “an utterly brillant rhetorician and visionary” with "modern-day appeal” who “provides the sacred space" for Americans to come together. Explanation for Incorrect Answer D : Choice (D) is Incorrect. Although, accoréing to the author, Jefferson's ideas accupy a “rarified” or elevated region, they also provide “a place where ordinary Americans can congregate to speak the magic words together.” The author does not imply that Jefferson's ideas are too obscure for the average person to understang The author would most ikely describe the “federal bureaucracies and taxes” (line 54) as WA) measures that mit some rights in order to safeguard other rights (©) excessive and unnecessary vielations of individual freedom, (©) examples of the kinds of governmental infringement that Jefferson could not foresee (©) proof that Jefferson’s words have an Influence on public policy today (© regutetions that reveal the true lack of freedom in America ANSWERS AND EXPLANATIONS: Explanation for Correct Answer A: Choice (A) is correct. The author uses the example of "federal bureaucracies and taxes” to illustrate the Issue of "opposite propositions” In Jefferson's claims. In the sentence In Wich this phrase appears, the author implie at, fone believes that Americans have FromSAT Online Course Sih CUUS AB ‘SAT 2006 May Answers and Explanations a right to health care and a clean environment, one should also believe in paying taxes and supporting federal bureaucracies, which are necessary to provide health care anc a clean environment. Jefferson's chetaric allows a person to believe in one but not the other, In this formulation, federal bureaucracies ane taxes are measures that limit some rights ("individual independence") In order to guarantee others ("health care anc 2 clean environment’) Explanation for Incorrect Answer B Choice (B) is incorrect, The author uses the exemple of "federal bureaucracies and taxes" to llustrate the Issue of "opposite propositions” in Jatferson's claims. In the sentence in which this phrase appears, the author implies that, if one believes that Americans have a right to health care and a clean environment, one should also believe In paying taxes and supporting federal bureaucracies, which are necessary to provide health care and 2 clean environment. The author might argue that federal bureaucracies and taxes are necessary, not unnecessary. Explanation fer Incorrect Answer C: Choice (C) is incorrect, The author calls Jefferson a “visionary.” A visionary is an Individual who has unusual foresight and imagination; by describing Jefferson as @ Vislonary, the author suggests thet Jefferson probably was aware of the future impalications of his claims and the limitations of individual rights. The author uses the example of “federal bureaucracies and taxes" to iustrate the Issue of “opposite propositions” in Jefferson's claims. Explanation for Incorrect Answer D: Choice (D) is incorrect. Although the author implies that Jefferson’s words influence Individuals’ thinking about rights, nowhere in the passage does the author discuss how Jefferson's ideas affect current public policy. The author uses the example of “federal tw illustrate the issue of “opposite propositions" bureaucracies and taxe: In Jefferson's Explanation for Incorrect Answer E Choice (E) is incorrect, The author uses the example of "Yederal bureaucracies and taxes" to llustrate the Issue of “opposite propositions” in Jefferson's claims. In the sentence in which this phrase appears, the author implies that, if one believes that Americans have @ right to health care and a clean environment, one should also believe In paying taxes and supporting federal bureaucracies, which are necessary to provide health care and 2 clean enviranment—even though these “violate inaividual Independence.” The author Is not discussing the "lack of freedom in America"; rather, the point is that freedom, in the real world, involves a series of trade-offs. FromSAT Online Course CUUS WABI ‘SAT 2006 May Answers and Explanations Sections: Writing Onlin ~ Practice Test #1 It being agreed by world leaders that there should be @ major organized effort to address poverty, illteracy, and disease; they then set goals by whic progress could be measured. (8) re being agreed by world leaders 18) world leaders agreed (©) World leaders who should agree (©) World leaders being in agreement (© World leaders, in agreement ANSWERS AND EXPLANATIONS Explanation for Correct Answar CChoice (B) Is correct. It avolds the error of the original by providing @ main verb agreed”) to carry out the action of the first independent clause ("World leaders aoreed .. . disease’) Explanation for Incorrect Answer A CChoice (A) is @ sentence fragment. There is no main verb to carry out the action of the first independent clause, only the participle “being.” Explanation for Incorrect Answer C: Choice (C) results in @ sentence fragment, There is no main verb to carry out of the first independent clause, only the relative clause "who should agree.” Explanation for Incorrect Answer D : Choice (D) results Ina sentence fragment. There Is no main verb to carry out the action of the first independent clause, only the participle “being.” Explanation for Incorrect Answer E Choice (E) results in a sentence fragment. There is no main verb te carry out the action of the first independent clause. FromSAT Online Course Sih CUUS HA ‘SAT 2006 May Answers and Explanations At lunchtime, Kevin paid for Anita's hamburger: the reason is because he owed he F money, ) amourger: the reason is because he owed (©) hamburger; this was because he owed (©) hamburger, in that ne owed 1) hamburger because he owed (©) hamburger becouse of owing ANSWERS AND EXPLANATIONS Explanation for Correct Answer D : Choice (D) is correct, It avoids the errors of the original by providing a concise dependent clause ("because he owed her money") to logically Indicate why “Kevin paid for Anita's hamburger.” Explanation for Incorrect Answer A: Choice (A) Involves unnecessary coordination and wordiness. There is no need to split the sentence into two independent clauses and use the redundant phrase “the reason is because.” Instead, the semicolon should be removed and the phrase "the reason is" eleted, so that @ dependent clause ("because he owed her money") can logically Indicate why "Kevin paid for Anita's hamburger.” Explanation fer Incorrect Answer Choice (B) involves unnecessary coordination and wordiness. There Is no need to split the sentence into two independent clauses and use the unnecessary phrase "this was because.” Instead, the semicolan should be removed and the phrase "this was” deleted, so that a dependent clause (*because he owed her money") can logically indicate why ‘Kevin paid for Anita's hamburger.” Explanation for Incorrect Answer C: CChoice (C) Is awkward and unidlamatic, The comma that follows “hamburger,” along with the phrase “in that,” should be deleted so that a dependent clause ("because he owed her money") can lagically indicate why "Kevin pald for Anita’s hamburger.” Explanation for Incorrect Answer E: Choice (E) is awkward and uniciomatic, The unidiomatic "because of owing” should be replaced with the precise, idiomatic phrase "because he owed.” Reading poetry aloud usually helps me understand it better. FromSAT Online Course Sih CUUS HA ‘SAT 2006 May Answers and Explanations W Reading poetry aloud ©) poetry, read aloud, (©) To read postry aloud, this (©) If one reads poetry aloud it (©) 1 read poetry aloud, it [ANSWERS AND EXPLANATIONS, Explanation for Correct Answer A CChoice (A) Is correct. It avoids the errors ofthe other options by providing a gerund Reading’) to indicate what “usually helps me understand it better” and provicing a ‘noun ("poetry") that can logically be a referent of the pronoun “it.” Explanation for Incorrect Answer B Choice (B) results in illogical pronoun reference, The only noun in the sentence that “it” can refer to is “Poetry.” However, it does not make sense to say that poetry helps one to understand poetry. Explanation for Incorrect Answer C+ Choice (C) results in faulty sentence structure, The pronoun “this” immeciately follows Its referent ("To read poetry aloud"), leaving the subject "To read poetry aloud” without 2 main verb Explanation for Incorrect Answer D: CChoice (D) results in a pronoun shift. The indefinite pronoun "one" is followed by the personal pronoun "me, Explanation for Incorrect Answer E Choice (E) results in @ comma splice. Two Independent clauses ("I read poetry aloud” ‘and “it usually helps me understand it better”) are joined by anly @ comma. One of the most beautiful trees In North America, the chestnut tree brought nearly ‘ko extinction by a fungus during the chestnut blight of 1909. () tree brought nearly to extinction by 2 fungus (© tree, which a fungus brought nearly to extinction (2) tree, but @ fungus brought it nearly to extinction w/©) tree was brought nearly to extinction by a fungus FromSAT Online Course Sih CUUS HA ‘SAT 2006 May Answers and Explanations (© trae, having been brought nearly to extinction by a fungus ANSWERS AND EXPLANATIONS: Explanation for Correct Answer D Choice (D)is correct. It avoids the sentence-fragment error of the original by providing {2 main verb ("was") to carry out the action of Explanation for Incorrect Answer A: CChoice (A) is @ sentence fragment, There is no main verb to carry out the action of the sentence, only the participle "brought, Explanation for Incorrect Answer B CChoice (B) results in a sentence fragment, There is no main verb to carry out the action of the sentence, only a relative clause ("which a fungus . . ). Explanation for Incorrect Answer C: Choice (C) results in faulty sentence structure. There is no main verb to carry out the action of the sentence, only @ subordinate clause (“but @ fungus...) Explanation for Incorrect Answer E 1 Choice (F) results in a sentence fragment. There is no main verb te carry out the action sentence, only a participa phrase ("having been brought . . .") A hot-air balloon rises when the air inside it becomes warm, drops when that air cools, and maves sideways when blown by the wind, 9) moves sideways when blown by the wine (©) moving sideways when the wind blows (2) wnen the wind blows, the movement is sideways. (©) caused to move sideways when blown by the wind (© itis blown by the wind, causing sideways movement ANSWERS AND EXPLANATIONS: Explanation for Correct Answer A: Choice (A) Is correct, It avoids the errors ofthe other options by providing @ verb (Cmoves") that is parallel to the two verbs that precede it rises” and "drops" Explanation for Incorrect Answer From SAT Online Course kth CUUS WAHL ‘SAT 2006 May Answers and Explanations Choice (B) results in faulty parallelism. A finite verb ("moves") is needed in place of the participle "moving" to be parallel to the finite verbs that precede It (*rses" and “érops") Explanation for Incorrect Answer C: Choice (C) results in multiple flaws in parallelism. The phrases that precede "when the wind blows, the movement Is sideways” consist of a finite verb ("rises" and “érops") followed by 2 relative clause Indicating time ("when the air inside it becomes warm” and “when th air cools"). Here the phrases are, Incorrectly, reversed, and the “the movement” has replaced the present-tense verb “moves.” Explanation for Incorrect Answer D Choice (D) results in faulty parallelism. The past-tense verb phrase “caused to move" Is nse verbs “rises” and “érops.” Explanation for Incorrect Answer E: CChoice (E) results in faulty parallelism, The independent clause “it is blown by the wind is not parallel to the present-tense verbs "rises" and “érops.” ‘The supply clerk was careless in taking inventory, its leading to a shortage of some items and an excess of others. (8) The supply clerk was careless in taking Inventory, ts leading (®) In taking inventory the supply clerk was careless this carelessness led (©) The supply clerk was careless in taking Inventory and ited ©) The supply clerk’s carelessness in taking inventory led (© The inventory, it was taken carelessly by the supply clerk, which led ANSWERS AND EXPLANATIONS Explanation for Correct Answer D Choice (D) is correct. It avoids the pronoun-reference error of the original by removing the possessive pronoun “its,” which does not lagleally refer to anything in the sentence. Explanation for Incorrect Answer A: CChoice (A) involves unclear pronoun reference. There is nothing in the sentence to Wich the possessive pronoun “its” can logically refer Explanation for Incorrect Answer B CChoice (B) results in @ comma splice. Two Independent clauses ("In taking inventory FromSAT Online Course Sih CUUS HA ‘SAT 2006 May Answers and Explanations careless" and "this carelessness led .. . of others") are joined by only a comma. Explanation for Incorrect Answer C: CChoice (C) Invelves unclear pronoun reference. There is nothing In the sentence to wiich the singular pronoun “it” can logically refer. Explanation for Incorrect Answer E CChoice (E) results in faully sentence structure. The pronoun “it awkwardly appears to make "it was taken ... clerk” an apposttve clause, which leaves the sentence without ‘2 main verb, only the appositive clause and a relative clause (which led... .”) In 1903, physicist Marie Curie was the first woman to win the Nobel Pris ‘moreover, in 1911, she became the first person to win it @ second time. (A) physicist larle Curle was the first woman to win the Nobel prim, moreover, (©) Marie Curie, a physlelst, won the Nobel prlae, which was the first time @ woman won i, then, ©) physicist Marie Curie became the first woman to win the Nobel Pris; (©) Marie Curie won the Nobel Priz for the frst time as a woman Who was @ physicist, and © physic larie Curie became the first winner of the Nobel Pris among women, ANSWERS AND EXPLANATIONS: Explanation for Correct Answer Cz Choice (C) is correct. It avoids the comma-splice error of te original by providing a semicolon to properly join two independent clauses ("In 1903, physicist... the Nobel e"). Pri and "in 2911, ... a secon Explanation for Incorrect Answer A: Choice (A) Is @ comma splice. Two Independent clauses ("In 1903, physicist... the Nobel Prix" and “moreover, in 1911, ... a second time") are joined by only a comma. Explanation for Incorrect Answer Choice (B) is awkward, wordy, and imprecise and results in @ comma splice. The phrase “won the Nobel Prize" and 1¢ awkwardly phrased relative clause “which was the First time a woman won it” should be shortened to read "became the first woman to win the Nobel Prize.” Also, to avoid the comma splice, the comma that follows "it" should be replaced by a semicolon. Furthermore, the word “then” is confusing and should be delet FromSAT Online Course Sih CUUS HA ‘SAT 2006 May Answers and Explanations ed. Explanation for Incorrect Answer Dz Choice (D) is awkward and Ilogical. It does not make sense to say that Marie Curle won the prize “Yor the first time as a woman.” She was always @ woman, not just when she won the prize. Explanation fer Incorrect Answer E CChoice (E) Is awkward and results In a comma splice, The awkward and unclear became the first winner of the Nobel Priz: among women” should be shortened to the more precise "became the first woman to win the Nobel Prize." In addition, two Independent clauses ("In 1903, physicist .. . among wamen” and "moreover, in 1911, 8 second time") are joined by only @ comma ‘Sometimes called “the founder of art in Texas,” a sculpture of Willam Jennings ‘Bryan was completed by Elisabet Nev in her Austin studio in 1899. (®) a sculpture of William Jennings Bryan was completed by Elisabet Ney in her Austin studio in 1899 (©) witiam Jennings Bryan's sculpture was completed by Elisabet Ney in her Austin studlo in 1899 (©) im her Austin stucio, Elisabet Ney having completed a sculpture of Willa Jennings Bryan in 1899 (©) because a sculpture of Willam Jennings Bryan was completed in 1899 in her ‘Austin studio by Elisabet Ney 9 Elisabet Ney completed a sculpture of William Jennings Bryan in her Austin studio in 1899 ANSWERS AND EXPLANATIONS Explanation for Correct Answer E : Choice (E) Is correct, It avolds the modification error of the original by providing a subject ("Elisabet Ney") that can be logically modified by the sentence's opening phrase “Sometimes calles the “founder of art in Texas."" Explanation for Incorrect Answer A: CChoice (A) involves logical modification, It does not make sense to say that “2 sculpture" Is "Sometimes called ‘the founder of art in Texas.” Explanation for Incorrect Answer FromSAT Online Course Sih CUUS HA ‘SAT 2006 May Answers and Explanations Choice (B) results in illogical madifieation. It daes not make sense to say that "William Jennings Bryan's sculpture” is "Sometimes called "the founder of art in Texas.”" Explanation for Incorrect Answer C: Choice (C) results in illogical modification and a sentence fragment. It does not make sense to say that Ney was “called "the founder of art ia Texas” only when “in her Austin studio." Also, the sentence has no main verb to carry out Its action, only the participa! phrase “having completed.” Explanation for Incorrect Answer D : Choice (D) resus Inilogical modification and a sentence fragment, A subject ("Elisabet Ney") that can be logically modified by the sentence’s opening phrase ("Sometimes called the ‘founder of art in Texas") Is needed in place of the dependent clause (because a sculpture... Ney") Professor Brand, who enjoys welcoming international students to her home on ‘Thanksgiving, served the traditional turkey dressed in Pilgrim clathing (8) served the traditional turkey dressed in Pilgrim clothing (©) served the traditional turkey, dressing in Pligrim clothing (©) serving the traditional turkey ané drassed in Pilgrim clothing V1) dressed in Pilgrim clothing to serve the traditional turkey (© dressed in Pilgrim clothing and the tracitional turkey was served ANSWERS AND EXPLANATIONS: Explanation for Correct Answer D : Choice (D) is correct. It avoids the modification error of the original by placing the phrase “dressed in Pilgrim clothing” so that it clearly applies to "Professor Brand.” Explanation for Incorrect Answer A Choice (A) invalves unclear modification. The placement of "dressed in Pilgrim clothing makes it unclear whether the phrase is meant to modify traditional turkey.” rofessor Branc” or “the Explanation for Incorrect Answer Choice (B) results in an awkward, unclear sentence. The participle “érassing" does not coordinate well with the past-tense verb “served,” resulting In a confusing, ungrammatical sentence. From SAT Online Course Skt CUUS WAHL ‘SAT 2006 May Answers and Explanations Explanation for Incorrect Answer C+ Choice (C) results in @ sentence fragment. There is no main verb to carry out the action of the sentence, only the participle “serving Explanation for Incorrect Answer E: Cchoice (E) results in faulty parallelism, What follows "dressed in Pilgrim clothing” should provide a past-tense verb to be parallel with “dressed.” Instead, an independent clause ("the traditional turkey was served") Is used Chop suey originated in the United States, and many people assume that It waschina, () and many people assume that it was (© many people assuming WE) not, a5 many people assume, in (©) not what many people assume being (© but many people assume it to be ANSWERS AND EXPLANATIONS: Explanation for Correct Answer Cz Choice (C) is correct. It avoids the errars of the original by providing “not” to clearly Ingicate the relationship between the ideas in the sentence and by avoiding unclear pronoun reference. The coordinat 19 conjunction “an” incicates two ideas of equal Importance; however, the sentence demands a clear contrast between the fact that “Chop suey originated in the United States” and what many people assume: that it originated "in China.” Also, the pronoun "It" in the original sentence does not have 8 logical referent. Explanation for Incorrect Answer A: Cchoice (A) involves improper coordination and unclear, logical pronoun reference. The coordinati 1 conjunction “anc” incicates two ideas of equal importance; however, the sentence demands a clear contrast between the fact that "Chop suey originated in the unites es" and what many people assume hat It originated "in China.” Furthermore, there is nothing in the sentence to which the pronoun “it” can logically refer. Explanation for Incorrect Answer Choice (B) results in faulty sentence structure. There is no main verb to carry out the FromSAT Online Course Sith CUUS WAIN ‘SAT 2006 May Answers and Explanations action of the second independent clause, ont 1¢ participle “assuming, Explanation for Incorrect Answer D: Choice (D} results In faulty sentence structure, The participle “belng” Is not appropriate ‘end of a complete relative clause (*not what many people assume") Explanation for Incorrect Answer E: Choice (E) results in unclear, logical pronoun reference. It is net clear whether the singular pronoun “it” Is meant to refer to the singular noun “Chop suey" or the singular oun "United States.” No matter, because neither can be logically referred to by “it” The discrepancy between the richness of Shakespeare's works and how much biographical information is lacking has not diminished over centuries. 9 how much biographical information is lacking (91 the tacking of much biographical information WC) the lack of biographical information about Shakespeare (©) there is lack of biographical information (© Shakespeare's biography ANSWERS AND EXPLANATIONS Explanation for Correct Anwar C: Choice (C) is correct. ts the only option that provides @ noun ("the lack of biographical information about Shakespeare") that logically completes the comparison iven by "The discrepancy between the richness of Shakespeare's works and...” Explanation for Incorrect Answer A: Choice (A) involves awkward and unclear phrasing, resulting in an logical comparison. The sentence Is meant to highlight the discrepancy between two things—"the richness of Shakespeare's works” and "the lack of biographical information about Shakespeare Instead, It highlights the discrepancy between the richness of Shakespeare's works and the amount of biographical information that is lacking (*how much biographical Information is lacking”) Explanation for Incorrect Answer B Choice (B) results In unidiomatic and imprecise phrasing, The phrase “the lacking of Is not idiomatic. in addition, the sentence does not make clear the fact that the “blogrephical information” being discussed is that of Shakespeare, FromSAT Online Course Siti CUUS WABI ‘SAT 2006 May Answers and Explanations Explanation for Incorrect Answer D: Choice (D) results in imprecise phrasing and faulty parallelism. The sentence is meant to highlight the discrepancy between two noun phrases. Instead, i provides only one noun phrase ("the richness of Shekespeare’s works”). In addition, the vague phrase hat the “there Is @ lack of biographical information” does not make clear the fa biographical information being discussed Is that of Shakespeare, Explanation for Incorrect Answer E: Choice (F) results in imprecise phrasing. Tt does not indicate what the other options do: that there is a lack of biographical information about Shakespeare. Global warming trends, what play a big role in the melting of tropical glaciers like those on Mt. Kilimanjaro, may cause many ice caps to vanish within 20 years, No ANSWERS AND EXPLANATIONS: Corrected Sentence: Global warming trends, which play a big role in the melting of tropical glaciers like those on Mt. Kilimanjaro, may cause many ice caps to vanish within 20 years. Explanation for Correct Answer A ‘The error in this sentence accurs at (A). The pronoun "what" is used only when there Is ro specific antecedent. In this sentence, there Is a specifi antecedent, “trends,” so the relative pronoun “whieh” is required, Explanation for Incorrect Answer ‘There is no error at (B). The gerund *melting” is appropriately introduced by the article tthe.” Explanation for Incorrect Answer C1 ‘There is no error at (C). The auxiliary verb “may” appropriately modifies the plural vero “cause,” which agrees with the plural subject “trends.” Explanation for Incorrect Answer D : FromSAT Online Course Sith CUUS WAIN ‘SAT 2006 May Answers and Explanations ‘There Is no error at (D). The preposition “within appropriately introduces the prepositional phrase “within 20 years," which appropriately modifies the verb “vanish,” Explanation for Incorrect Answer E : There is an error in the sentence. Charles Dickens’ Great Expectations 's a novel in which the main character becomes so obsessed with the idea of becoming a gentleman that his moral judgment is temporarily affected, No errar ANSWERS AND EXPLANATIONS: Corrected Sentence: Charles Dickens’ Graat Expectations is a novel in which the main character becomes so obsessed with the idea of becoming a gentleman that his moral judgment is temporarily affected. Explanation for Correct Answer E : There is no error inthis sentence Explanation for Incorrect Answer A: ‘There Is no error at (A), The preposition “in” appropriately introduces the prepositional phrase “in which the main character becomes so obsessed," which modifies the noun “novel The relative pronoun "which" refers to the noun “novel” Explanation for Incorrect Answer ‘There is no error at (B). The past participle “obsessed” complements the noun “character,” ané the preposition “with” idlamaticaly follows "obsessed" and appropriately introduces the prepositional phrase “with the idea.” Explanation for Incorrect Answer C1 ‘There is no error at (C). The prepositional phrase “of becoming” appropriately modifies the noun “idea.” Explanation for Incorrect Answer D: ‘There is no error at (0), The subordinating conjunction “that” appropri ly serves to link the clauses “the main character becomes so obsessed” and *his moral judgment is temporarily affected.” From SAT Online Co urse i CLUS ABI ‘SAT 2006 May Answers and Explanations The candidate called for medical insurance reform, but to me he seemed less Interested in the plight of uninsured citizens than in whether enough ig registeredt vote. No error ANSWERS AND EXPLANATIONS: Corrected Sentence: The candidate called for medical insurance reform, but to me he seemed less interested in the plight of uninsured citizens than in whether enough are registered to vote. Explanation for Correct Answer Cz ‘The error inthis sentence accurs at (C). The singular verb "is" does not agree with the plural subject “enough” (short for “enough uninsured citiens”) and should be replaced with the plural verb “are.” Explanation for Incorrect Answer A: ‘There is no error at (A). The past-tense verb “called” is appropriate in this context, and the preposition “for” appropriately introduces the prepositional phrase “for medical insurance reform. Explanation for Incorrect Answer B There Is no error at (8). The subordinating conjunction "than" correctly introduces the adverbial clause “than In whether enough [are] registered to vate.” Explanation for Incorrect Answer D : ‘There Is no error at (D). The subject complement “registered” correctly describes the oun “enough” and appropriately follows the linking verb. Explanation for Incorrect Answer E 1 There is an error in the sentence. Our new neighbors are the most sociable people we have ever met, and ourchief Interests, cooking and politics, are similar to them. No error FromSAT Online Course Siti CUUS WABI ‘SAT 2006 May Answers and Explanations ANSWERS AND EXPLANATIONS: Corrected Sentence: Our new neighbors are the most sociable people we have ever ret, and our chief interests, cooking and poltcs, are similar to theirs. Explanation for Correct Answer D : The error in this sentence occurs at (0). The pronoun “them” creates an improper camparison between “our chief interests” and “Our new neighbors” and should be replaced with "theirs," a pronaun representing their (the new neighbors’) interests. Explanation for Incorrect Answer A: ‘There Is no error at (A), The superlative "most” appropriately modifies the adjective “sociable Explanation fer Incorrect Answer B ‘There Is no error at (B). The adverb “ever” appears in the middle of the verb phrase “have ever met” and appropriately modifies the verb "met." In addition, the plural verb “have” agrees with the plural subject "we." Explanation for Incorrect Answer C+ There Is no error at (C). The adjective “chief” appropriately modifies the noun interests.” Explanation for Incorrect Answer E: There is an error in the sentence, Katherine feft that she has not had any understanding of thehighly intricate workings of the stock market until her uncle took her to the New York Stock Exchange. No error ANSWERS AND EXPLANATIONS: Corrected Santence: Katherine felt that she had not had any understanding of the highly intricate workings of the stock market until her uncle took her to the New York Stock Exchange. FromSAT Online Course Siti CUUS WABI ‘SAT 2006 May Answers and Explanations Explanation for Correct Answar A: ‘The error in this sentence accurs at (A). The past perfect tense ("had not had”) is needed to indicate an action that ended in the past. Explanation for Incorrect Answer B ‘There Is no error at (B). The adjective “any” Is used correctly to modify the noun ~understanging,” and the preposition “of” appropriately introduces the prepositional phrase “of the... workings.” Explanation for Incorrect Answer C: ‘There Is no error at (C). The aéverb “highly” is used correctly to modify the adjective icate," which in turn is used correctly to modify the noun "workings." Explanation for Incorrect Answer D : ‘There is no error at (0). The preposition “until” appropriately begins the prepositional phrase “unt her uncle took her.” Explanation for Incorrect Answer E: There is an error in the sentence. that the police officer arrived quickly because she was the only person at the scene who was able to investigate the accident calm and dispassionately. No error ANSWERS AND EXPLANATIONS. Corrected Sentence: It was fortunate that the police officer arrived quickly because she was the only person at the scene who was able to Investigate the accident calmly and dispassionately. Explanation for Correct Answer D : ‘The error in this sentence occurs at (D). The verb "investigate" is incorrectly modified by an adjective, “calm.” The acverb “calmly” should be used. Explanation for Incorrect Answer A: FromSAT Online Course S&{h CUUS HERS, ‘SAT 2006 May Answers and Explanations ‘There is no error at (A). The singular linking verb *was” agrees with the singular subject “It” and links the subject to the complement “fortunate.” Explanation for Incorrect Answer ‘There Is no error at (B). The subordinating conjunction "because” appropriately was fortunate that the police officer arrived Quickly." The pronoun *she* appropriately refers to the police officer, and the singular Ingicates a reason forthe first clause, “ verb *was" agrees with the singular subject *she.” Explanation for Incorrect Answer C : ‘There Is no error at (C). The infinitive verb “to Investigate” appropriately serves as the irect abject of the verb phrase “was able.” Explanation for Incorrect Answer E There is an error in the sentence. During rehearsals, the director praised the actors which had supporting roles more often than those with the most demanding roles. No error ANSWERS AND EXPLANATIONS. Corrected Sentence: During rehearsals, the director praised the actors who had supporting roles more often than those with the most demanding roles. Explanation for Correct Answer At ‘The error inthis sentence occurs at (A). The relative pronoun “which” should be used to refer to animals, things, ideas, and qualities (not to people) and should be replaced with the relative pronaun “who.” Explanation for Incorrect Answer ‘There is no error at (B). The past-tense verb “had” is appropriate in this sentence, as Incicatee by the past-tense vero “praised.” Explanation for Incorrect Answer C ‘There is no error at (C). The subordinating conjunction "than" appropriately introduces the abbreviated adverbial clause "than those with the most demanding roles.” From SAT Online Co urse i CLUS ABI ‘SAT 2006 May Answers and Explanations Explanation for Incorrect Answer D: ‘There is no error at (0). The superlative *most” appropriately modifies the adjective “demanding,” which modifies the noun “roles. Explanation for Incorrect Answer E : There is an error in the sentence. What becomes apparent from assessing scientific accomplishments is that only relatively recently have the necessary technology been developed for solving the mysteries of genetics. No error ANSWERS AND EXPLANATIONS: Corrected Sentence: What becomes apparent from assessing scientific accomplishments is that only relatively recently has the necessary technology been developed for solving the mysteries of genetics, Explanation for Correct Answer C: ‘The error inthis sentence accurs at (C). The plural verb “have” does not agree with the delayed singular subject has." technology” and should be replaced with the singular verb Explanation for Incorrect Answer A: ‘There is no error at (A). The pronoun "What" is the appropriate relative pronoun to use Ina sentence where there 's ne antecedent for the pronoun, Explanation for Incorrect Answer B There Is no error at (B). The preposition “from” appropriately Introduces the prepositional phrase “from assessing scientific accomplishments,” which modifies the adjective “apparent reposition “fram.” ‘The gerund “assessing” ppropriately serves as the object of the Explanation for Incorrect Answer D : ‘There is no error at (D). The linking verb "been" appropriately links the subject “technology” with the complement “developed.” The preposition "Yor" appropriately Introduces the prepositional phrase "Tor solving the mysteries,” which modifies the ver FromSAT Online Course Sith CUUS WAIN ‘SAT 2006 May Answers and Explanations phrase “been developed Explanation for Incorrect Answer E: There is an error in the sentence. Because his experience in the naval medical corps had been rewarding, Bobapplled ‘to medical schoo! after he was discharged from the navy. No error ANSWERS AND EXPLANATIONS: Corrected Sentence: Because his experience in the navel medical corps had been rewarding, Bob applied to medical school after he was discharged from the navy. Explanation for Correct Answer E = There is no error in this sentence, Explanation for Incorrect Answer A: ‘There is no error at (A). The subordinating conjunction "Because" appropriately links. the subordinate clause "Because his experience in the naval medical corps had been rewarding” to the main clause "Bob applied to mecical school ater he was discharged from the navy.” Explanation for Incorrect Answer There is no error at (B). The participle "rewarding" appropriately modifies the noun experience." Explanation for Incorrect Answer C ‘There is no error at (C). The past-tense verb “appli Is appropriate in the context of the sentence, The preposition *to”idlomatically follows the verb “applied” anc appropriately intraduces the prepositional phrase "to medical school.” Explanation fer Incorrect Answer D : ‘There is no error at (0). The singular pasttense verb phrase "was dischargec” appropriately describes an action by the singular subject (Bob) that took place in the past. Ocean currents that start in the warm waters of the Pacific and Indian Oceans arei FromSAT Online Course Sith CUUS AIK ‘SAT 2006 May Answers and Explanations nitially [ow in salt content but become more saline as it flows northward Into colder regions. No error ANSWERS AND EXPLANATIONS Corrected Sentence: Ocean currents that start in the warm waters o Pacific and Indian Oceans are initially low in salt content but become more saline as they flow northward into colder regions: Explanation for Correct Answer D : ‘The error in this sentence occurs at (D). The singular pronoun “it” incorrectly refers to the plural subject “currents” and should be replaced with the plural pronoun “they. In ‘addition, the singular verb “laws” should then be replaced with plural vero “flow.” Explanation fer Incorrect Answer A: ‘There Is no error at (A). The proper nouns “Pacific and indian Oceans" are ‘appropriately introduced by the article “the.” Explanation for Incorrect Answer ‘There is no error at (B). The adverb “initally” appropriately modifies the adjective low." Explanation for Incorrect Answer C ‘There Is no error at (C). The plural Inking verb "become agrees with the plural subject “currents” and appropriately Iinks the subject with the subject complement “saline,” The adverb "more" appropriately madifies “saline.” Explanation for Incorrect Answer E 1 There is an error in the sentence. Plants grown for their flowers should be treated with 2 low-nitrogen fertilizer because excessive nitrogen promotes the growth of leaves at the expense offlowers. No error FromSAT Online Course Sih CUUS AB ‘SAT 2006 May Answers and Explanations ANSWERS AND EXPLANATIONS Corrected Sentence: Plants g-own for their flowers should be treated with 2 low nitrogen fertilaer because excessive nitrogen promotes the growth of leaves at the expense of flowers. Explanation for Correct Answer Ez There is no error in this sentence, Explanation for Incorrect Answer A: ‘There is no error at (A), The past participle “grown” appropriately mocifes the noun Plants. The preposition "fer" appropriately thelr flowers,” which modifies "grown." duces the prepositional phrase “for Explanation for Incorrect Answer ‘There is no error at (B). The preposition "with" idiomatically follows the verb phrase "be treated” and appropriately introduces the prepositional phrase “with a low-aitrogen fertilizer,” which modifies the verb phrase "should be treated.” Explanation for Incorrect Answer C ‘There Is no error at (C). The singular verb "promotes" agrees with the singular subject "gen." Explanation for Incorrect Answer D: ‘There Is no error at (0), The preposition “at” appropriately introduces the prepositional phrase “at the expense,” which modifies the verb “promotes.” The preposition “of” appropriately Intraduces the prepositional phrase “of flowers,” which mocifies ‘expense. ‘Those who defend sequoia trees from loggers justified doing so on the grounds that such trees are ireplaceable. No error ANSWERS AND EXPLANATIONS Corrected Sentence: Those who defend sequoia trees from loggers justify doing so on the grounds that such trees are irreplaceable. FromSAT Online Course Sih CUUS HA ‘SAT 2006 May Answers and Explanations Explanation for Correct Answer C : ‘The error in this sentence accurs at (C). The verb “justified” is in the incorrect (past) tense, The verb should be in the present tense, as indicated by the present-tense verb “defend Explanation for Incorrect Answer At ‘There Is no error at (A). The demonstrative pronoun “Those” refers to the relative pronoun “who,” which connects “Those” to the subordinate clause "who defend sequola Explanation for Incorrect Answer ‘There is no error at (B). The preposition “from” appropriately intaduces the prepositional phrase “from loggers." Explanation for Incorrect Answer D: ‘There Is no error at (0), The gerund “doing” is the direct object of the verb "Justify, and the adverb "so" modifies “doing.” Explanation for Incorrect Answer E: There is an error in the sentence. Extending along several city blocks are a row of ginkgo trees, their leaves turnings brillant yellow now that summer is over. No error ANSWERS AND EXPLANATIONS Corrected Sentence: Extending along several city blocks is a row of ginkgo trees, their leaves turning a brillant yellow now that summer is over. Explanation for Correct Answer B ‘The error in this sentence occurs at (B). The plural verb “are” does not agrae with the delayed singular subject “row” and should be replaced with the singular verb “is.” Explanation for Incorrect Answer A: ‘There is no error at (A). The present participle "Extending" appropriately modifies the FromSAT Online Course Sih CUUS AB ‘SAT 2006 May Answers and Explanations Explanation for Incorrect Answer C: ‘There is no error at (C). The plural possessive pronoun “their” appropriately takes the place of the plural noun “trees,” and the plural noun “leaves” indicates what is being possessed, Explanation for Incorrect Answer Dt ‘There Is no error at (D). The vers “turning” appropriately describes a continuous action by the leaves. Explanation for Incorrect Answer E : There is an error in the sentence. Africa's Kanem empire, after enduring for over a thousand yeers, is believed to have fallen into decine when trade centers shift outside its boundaries. No error ANSWERS AND EXPLANATIONS Corrected Sentence: Africa's Kanem empire, after enduring for over a thousand years, is believed to have fallen into decline when trade centers shifted outside its boundaries Explanation for Correct Answer D : The error in this sentence occurs at (D). The tense of the verb “shit” Is incorrect, The past tense “shifted” is required, as indicated by the past-tense verb phrase *have fallen.” Explanation for Incorrect Answer A: ‘There is no error at (A). The verb “enduring” appropriately describes @ continuous action by the subject “empire,” and the prepasition “Yor” appropriately intraduces the prepositional phrase “Yor over a thousand years," which modifies the verb “enduring.” Explanation for Incorrect Answer ‘There is no error at (B). The singular verb "is" agrees with the singular subject “empire.” Explanation for Incorrect Answer C: ‘There Is no error at (C). The preposition “into” appropriately introduces the preposition FromSAT Online Course Sih CUUS AB ‘SAT 2006 May Answers and Explanations al phrase “into dectine.” This prepositenal phrase idiomaticaly follows the verb phrase nave fallen.” Explanation for Incorrect Answer E : There is an error in the sentence. ‘Opposite to most people 1 know, Annie, @ good photogrepher herself, actuallyenjoys seeing the photographs that her friends take on their vacations, No ANSWERS AND EXPLANATIONS Corrected Sentence: Unlike most people I know, Annie, @ goad photographer herself, actually enjoys seeing the photographs that her friends take on their vacations. Explanation for Correct Answer A: The error in this sentence occurs at (A). The phrase "Opposite to” is not idiomatic in this context. Instead, the word "Unlike" is needed so that the intended comparison between *most people I know” and “Annie” is made clear, Explanation for Incorrect Answer ‘There Is no error at (B), The reflexive pronoun "herself" appropriately refers to the subject "Annie." Explanation fer Incorrect Answer C : There is no error at (C). The singular verb “enjoys” agrees with the singular subject “Annie,” and the gerund *seeing” describes what Annie enjoys. Explanation for Incorrect Answer D ‘There is no error at (0). The preposition “on” appropriately intraduces the prepositional phrase “on their vacations,” which modifies the verb "take." The possessive pronoun ‘their’ appropriately refers to the friends of Annie Explanation for Incorrect Answer E: There is an error in the sentence, FromSAT Online Course Sith CUUS AIK ‘SAT 2006 May Answers and Explanations My roommate, a drama major, claimed that Ibsen’s plays, unlike lonesco, aretotally conventional in their style. No error ANSWERS AND EXPLANATIONS Corrected Sentence: My roommate, a drama major, claimed that Ibsen's plays, Unlike 1onesco's, are totelly conventional in teir style, Explanation for Correct Answer B “The error inthis sentence accurs at (8). The phrase “unlike Ionesco” creates an iogical comparison between Ibser's plays and the playwright Ionesco. In order to show a ‘comparison between Tbser’s plays and lonesco's plays, “unlike Tonesco" should be replaced with the possessive phrase “unlike Ionesco’s.” Explanation for Incorrect Answer A : ‘There Is no error at (A). The past-tense verb “claimed” indicates an actlon taken by the subject “roommate” in the past, and the pronoun “that” functions as the direct object of the verb “claimed.” Explanation fer Incorrect Answer C : ‘There is no error at (C). The adverb “totaly” appropriately modifies the adjective "conventional," which modifies the noun “plays.” Explanation for Incorrect Answer D : ‘There Is no error at (0). The possessive pronoun “their” appropriately refers to the plays. Explanation for Incorrect Answer E1 There is an error in the sentence. Not very particular in nesting sites, house wrens may nest in birdhouses, ‘mailboxes, building crevices—even in the pockets of hanging laundry. No error ANSWERS AND EXPLANATIONS. FromSAT Online Course kh CUUS HERS, ‘SAT 2006 May Answers and Explanations Corrected Sentence! Not very particular about nesting sites, house wrens may nest in birdhouses, mailboxes, building crevices—even in the pockets of hanging laundry Explanation for Correct Answar A: ‘The error in this sentence occurs at (A). The preposition “in” is unidlomatic after articular” inthis context. The appropriate preposition to follow "particular" is “about, Explanation for Incorrect Answer B There is no error at (B). The plural noun “sites” is an appropriate word to indicate possible locations for nest building, Explanation for Incorrect Answer C: ‘There is no error at (C). The auxiliary verb "may" appropriately modifies the plural verb “nest,” which agrees with the plural subject "house wrens.” Explanation for Incorrect Answer Dt ‘There is no error at (0). The prepasition “of” appropriately introduces the prepositional phrase “of hanging laundry," which appropriately modifies the noun “pockets.” Explanation for Incorrect Answer E : There is an error in the sentence. [At the reception ware the chattering guests, the three-tiered cake, and the lively music that have become characteristic of many wedding celebrations, No error ANSWERS AND EXPLANATIONS Corrected Sentence: At the reception were the chattering guests, the three-tiered cake, and the lively musle that have become characteristic of many weeding celebrations: Explanation for Correct Answar Ez There is no error in this sentence, Explanation for Incorrect Answer A: “There is no error at (A). The preposition "At" appropriately introduces the prepositional phrase “At the reception," which modifies the verb “were. FromSAT Online Course Sih CUUS HA ‘SAT 2006 May Answers and Explanations Explanation for Incorrect Answer ‘There is no error at (B). The plural verb “were” agrees with the plural subject “guests, cake, and... music Explanation for Incorrect Answer C: ‘There Is no error at (C). The present participle “chattering” appropriately modifies the noun “guests.” Explanation for Incorrect Answer Dz ‘There is no error at (D). The subject complement “characteristic idlomatically folows the linking verb phrase "have become” and appropriately describes the subjects. The preposition “of” idiomaticaly fellows the adjective “characteristic” and appropriately Introduces the prepositional phrase “of many wedding celebrations.” (2) History isthe study not only of cataclysmic events but also of seemingly insignificant things, like the chill pepper, (2) Thechill pepper was known only in the Americas. (3) When Columbus ran into the chil pepper, he mistakenly thought It was related to black pepper, which was widely traded in Europe. (4) Once Columbus introduced the chili pepper to Europe, it spread quickly throughout Europe and Asia. (5) The introduction of the chili pepper had a tremendous impact on the balance of power among the European countries that were already Involved in the black pepper trade, and, what was even larger in scope, on Asian and European cultures. (6) Before the discovery of chill pepper, black pepper was considered so valuable that it was the focus of international rivalry. (7) Egypt and the Italian city-states, Venice and Genoa, wanted to retain exclusive control of the black pepper trade and the economic power that went along with It. (8) Spain's explorers went to sea, and one reason for it was black pepper. (9) They wanted to get it by bypassing the trade route controlled by the Arabs ‘and Italians, (10) But instead of black pepper, they found the chili pepper and discovered that it had many advantages over black pepper. (11) Among them was that it was a lot easier to grow than black pepper: it could grow in cool regions as well as hot ones In context, which should be added to the beginning of sentence 2? FromSAT Online Course kh CUUS HERS, ‘SAT 2006 May Answers and Explanations (®) Because it was unknown in Europe and Asia, (®) To the astonishment of most people, ©1©) Until the expecitions of the European explorer, Columbus, (©) Moreover, after the years of European exploration (©) why chill peppers have turned out to be so important is that ANSWERS AND EXPLANATIONS Explanation for Correct Anawar C: Choice (C) is correct. Sentence 2 states, ‘The chill pepper was known only in the “Americas.” This statement refers to the past, but exactly what point in the past Is Unclear. Choice (C) indicates the tme as well as introduces Columbus, who will be the focus of rest of the paragraph. Explanation for Incorrect Answer A: Choice (A) is unsatisfactory. Sentence 2 states, “The chill pepper was known only in the Americas. Beginning the sentence with the phrase "Because it was unknown in Europe and Asia” implies that the chill pepper was known only in the Americas as a result ofits being unknown in Europe and Asia, which is redundant and unhelpful. In addition, the beginning phrase gives no context in time for when chil peppers became known throughout the worl. Explanation for Incorrect Answer Bt Choice (B) is unsatisfactory. Beginning sentence 2 with the phrase "To the astonishment of mast people” implies that most people would be surprised to know that at some point the chili pepper was known only in the Americas. There is no way to Getermine if most people would be astonished, and the fact that the chili pepper was conce knowin only in the Americas is not the focus of the paragraph, so even if most people were astonished, it would not be relevant. Explanation for Incorrect Answer D : Choice (b) is unsatisfactory. The second paragraph explains how Columbus, @ European explorer, introduced the chill pepper to Europe and until that time, the chill pepper was known only In the Americas. Choice (0) states that the chili pepper was known only in the Americas “after the years of European exploration,” which does not make chronological sense. Explanation for Incorrect Answer E: Choice (E) Is unsatisfactory, The fact that at one point the chill pepper was known only Inthe Americas has litle to do with its importance. The third paragraph indicates that FromSAT Online Course CUUS WABI ‘SAT 2006 May Answers and Explanations the chill pepper’s importance stems from the effect It had on the balance of power ‘among European countries Involved in the black pepper trade and the Impact it had on the cultures of Asia and Europe. Which is the best replacement for “ran into" in sentence 3? W) encountered (© met up with (©) observee ©) contro © got hold of ANSWERS AND EXPLANATIONS: Explanation for Correct Answer A: Choice (A) is correct. Replacing "ran into” with “encountered” creates the sentence, "When Columbus encountered the chill pepper, he mistakenly thought it was related to black pepper, which was widely traded in Europe.” This sentence indicates that Columbus is seeing the chil pepper for the first time, and “encountered” Is @ more precise, appropriately formal word choice Explanation for Incorrect Answer Choice (B) is unsatisfactory. The phrase “met up with" is a casual term usually meaning seeing an acquaintance, Sentence 3 describes Columbus's encounter with the chil pepper, not a person, so "met up with" is nat appropriate Explanation for Incorrect Answer C+ CChoice (C) is unsatisfactory. The word "observed" implies viewing without interaction, It's logical that Columbus did mare than observe the chili pepper if he concludes that it was related to black pepper and he introduced it to Europe. Explanation for Incorrect Answer D: Choice (D) is unsatisfactory. To confront means to come face ta face with, especially with hostilty or a predetermined purpose. Its ilagical that Columbus confronted the chil pepper. Explanation for Incorrect Answer E Choice (F) Is unsatisfactory. The phrase "got holé of” implies that the chili pepper was s FromSAT Online Course Sih CUUS HA ‘SAT 2006 May Answers and Explanations lomething Columbus desired and finally obtained. Since the chili pepper was unknown to Columbus until his first encounter with it In the Americas, he could nat have been hoping to obtain it. In addition, the essay requires a more formal wore choice than the casual phrase “got hold of.” Of the following, which is the best revision for the underlined part of sentence 5 (reproduced below)? The introduction of the chil pepper had a tremendous impact on the balance of power among the European countries that were already Involved in the black pepper trade. and. what was even larger in scope, on Asian and European cultures, W trade, not to mention on (©) trade, but also (©) trade, an impact also on (©) trade; it also was affecting (© trade, and they also affected ANSWERS AND EXPLANATIONS Explanation for Correct Answer A : Choice (A) Is correct. In the context of sentence 5, the phrase “not to mention” shows @ relationship between the impact the chili pepper had on the balance of power in European countries and the even larger issue of the Impact the chill pepper had on the cultures of Asia and Europe. The revised sentence would read, "The introduction of the chill pepper had 2 tremendous impact on the balance of power among the European countries that were already involved in the black pepper trade, not to mention on Asian and European cultures. Explanation for Incorrect Answer Choice (B) is unsatisfactory. The phrase “but also” should be preceded with the phrase “nat only," as in “The introduction of the chili pepper had a tremendous impact on not only the balance of power..." The phrase “not only" does not appear in sentence 5; therefore, the phrase “but also” is not idiomatic, Explanation for Incorrect Answer C CChoice (C) is unsatisfactory. The phrase “an impact also on" does not establish the relationship between the impact of the chil pepper on the balance of power in Europea From SAT Online Co arse i CLUS HAL ‘SAT 2006 May Answers and Explanations countries and the larger issue of the Impact on Asian and European cultures. In addition, revising sentence 5 this way creates an awkward and unclear sentence, Explanation for Incorrect Answer D: Choice (D) is unsatisfactory. Combining the two concepts in sentence 5 (the impact on the balance of power in European countries and the impact cultures) with a semicolon (trade; It also was affecting”) does not establish the n Asian and European relationship of smaller to larger issues. In addition, the past progressive tense of ‘was affecting” is not parallel with the past tense In the rest of sentence 5. Explanation for Incorrect Answer E Choice (E) Is unsatisfactory. Since the subject of sentence 5, “The introduction of the chill pepper," Is singular, there is nothing in the sentence to which the pronoun "they" can logically refer. Which of the following would most improve the third paragraph (sentences 5-7)? W/) A more detalled discussion of how the introduction of the chili pepper influenced Asian and European cultures (©) An account of earter attempts to take control ofthe black pepper trade away from Egypt and the Italian city-states, (©) More information about how black pepper came to be so expensive (©) list of the countries that were affected by the introduction ofthe chili pepper (© A comparison of the price af black pepper in diferent countries in Europe during the Middle Ages ANSWERS AND EXPLANATIONS Explanation for Correct Answer A: CChoice (A) Is correct, Sentence 5 states that the chili pepper had a “tremendous Impact” on the cultures of Asia and Europe. More Information en this tople would strengthen this statement and the thied paragraph as @ whole. Explanation for Incorrect Answer B CChoice (B) is unsatisfactory. The focus of the paragraph is the impact of the chil pepper on the black pepper trade and, even more, on Asian and European cultures. A etalled discussion of attempts to take control ofthe black pepper trade away from Egypt and the Italian city-states (witho ‘ference to the chill pepper) would not From SAT Online Co rset CUUS AB i ‘SAT 2006 May Answers and Explanations Explanation for Incorrect Answer C: CChoice (C) Is unsatisfactory. The focus of the paragraph is the impact of the chill pepper on the black pepper trade and, even more, on Asian and European cultures White itis important to note that black pepper was valuable, itis nat necessary to discuss the economic history of black pepper in detall Explanation for Incorrect Answer Dt Choice (D) is unsatisfactory. While the focus of the passage and the paragraah is the Impact of the chill pepper on the black pepper trade and, even more, on Asian and European cultures, it is not necessary to list each country affected. Discussing key countries ané continents is sufficient. Explanation for Incorrect Answer E: Choice (E) Is unsatisfactory. While it's important to note that black pepper was valuable in Europe atthe time the chili pepper was introduced to the continent, itis not necessary to list the prices of black pepper in different countries in Europe. It is sufficient to state that black pepper was valuable In general In context, which is the best way to combine sentences 8 and 9 (reproduced below)? ‘Spain's explorers went to sea, and one reason for it was black pepper. They wanted to get it by bypassing the trade route controlled by the Arabs and Italians. Spain's explorers went to sea, and one reason for that was they wanted to bypass the trade route controlled by the Arabs and Italians for black pepper. (®)spain’s explorers went to sea, one reason for this was to find black pepper, bypassing the trade route controlled by the Arabs and Italians. (CiFor black pepper, Spain's explorers went to sea, one reason for which was to bypass the trade route controlled by the Arabs and Italians. (Because the Arabs an¢ Italians controlled the trade route of black pepper, Spain's explorers went to sea as a bypass. one reason that Spain's explorers went to sea was to get black pepper by bypassing the trade route controlled by the Arabs and Italians. ANSWERS AND EXPLANATIONS FromSAT Online Course Sih CUUS HA ‘SAT 2006 May Answers and Explanations Explanation for Correct Answer E Choice (F) is correct. This method of combining sentences 8 and 9 produces a sentence that Is clear and grammatically correct, It indicates that getting black pepper by bypassing the trade route controlled by Arabs and Italians was one of the reasons that Spain's explorers went to sea. Explanation for Incorrect Answer A: Choice (A) Is unsatisfactory. Placing “for black pepper” at the end of the combined sentence creates a confusing result. This method of combining indicates that the Arabs and Italians controlled the trade route in order to gain black pepper, and this is not the original Intent of sentences 8 and 8. Explanation for Incorrect Answer Choice (B) is unsatisfactory. This method of combining sentences 8 and 9 produces @ grammatically incorrect sentence, Removing the “and from sentence 8 has created a comma splice, as the independent clauses *Spain’s explorers went to sea” and “one reason for this was ..." are now joined only by a comma Explanation for Incorrect Answer C+ CChoice (C) is unsatisfactory. This method of combining sentences 8 and 9 awkwardly separates the concept of gaining black pepper from the concept of bypassing the trade routes controlled by the Arabs and Italians, thereby making the relationship between these two concepts confusing and unclear. The ariginal intent of sentences 8 and 9s to present these two concepts as components of one reason Spain's explorers went to sea. Explanation for Incorrect Answer D CChoice (D) is unsatisfactory. This method of combining sentences 8 and 9 creates @ sentence that implies black pepper owned a trade route. People, such as the Spanish, Arabs, or Italians, can own a trade route, but black pepper cannot. Also, the essential point that Spain's explorers wanted to get black pepper is stated in an indirect, awkward manner. Which is the best sentence to put after sentence 11? (8) to the United States you can even grow chil peppers in northern states like Minnesota W) Suddenly, there was a hardy alternative to black pepper. FromSAT Online Course Sih CUUS AB ‘SAT 2006 May Answers and Explanations (110 the eng, the Europeans never found a passage to the spice countries through the Amecican continents (©) Black pepper is still popular today, and much less expensive than it was a few centuries age (© The majority of peppers are grown and consumed in Mexico and Latin America ANSWERS AND EXPLANATIONS. Explanation for Correct Answer B Choice (B) is correct. The fourth paragraph describes how the chill pepper affected the black pepper trade. Indicating that the chill pepper became an “alternative to black pepper” effectively describes the impact of the chill pepper. Explanation for Incorrect Answer At Choice (A) is unsatisfactory. While choice (A) seems to elaborate on the concept of sentence 11 (growing chill peppers), the passage as @ whole Is about chill peppers and their historical effect on the black pepper trade. Its out of context to discuss growing chil peppers in the northern United States. Explanation for Incorrect Answer C+ CChoice (C) is unsatisfactory. The passage is about chill peppers and thelr historical effect on the black pepper pepper (and the Americas) in the process of trying to find @ new trade route, their failure to find this trade route is not the focus of the paragraph. .de. While Europeans may have encountered the chil Explanation for Incorrect Answer D Choice (D) is unsatisfactory. The focus of the passage is chili peppers and their historical effect on the black pepper trace. To describe this effect, sentence 11 notes the advantages the chili pepper had over black pepper. It would not be logical to follow sentence 11 with 2 statement about the current popularity or price of black pepper, Explanation for Incorrect Answer E CChoice (E) Is unsatisfactory. While sentence 21 is about growing chili pepzers, the focus of the passage is the chili pepper’s effect on the black pepper trade at the time of Columbus. It is outside the context of the passage and paragraph to follow sentence 11 With information about where chill peppers are currently grown and consumed. Section8: Critical Reading Online - Practice Test #1 FromSAT Online Course Sih CUUS AB ‘SAT 2006 May Answers and Explanations ‘The outbreak of disease could not be labeled pandemic, since despite its rapid: ~ it remained confined to one small region. W) spread (© concentration (2) fading (©) symptoms © improvement ANSWERS AND EXPLANATIONS: Explanation for Correct Answer A Choice (A) is correct, In this context, “spread” means to be extended or distributed, ‘The sentence indicates that the outbreak of disease could not be labeled “pandemic,” or ‘occurring aver a wide geographic area and affecting an exceptionally high proportion of the population. The ward “despite” in the second half of the sentence suggests that there will be a contrast between the missing word and the phrase “it remained confined to one small region.” Although the cisease spread, or extended, quickly, it could not be 1d to one small area considered pandemie because it was stl limi Explanation for Incorrect Answer B Choice (B) is incorrect, In this context, “concentration” describes an amount of something in a given area. The sentence indicates that the outbreak of disease could rot be labeled "pandemic," or occurring over @ wide geographic area and affecting an exceptionally high proportion of the population. The word “despi of the sentence suggests that there will be a contrast between the missing word and the phrase “it remained confined to one small region.” Since “concentration” is similar In meaning to the phrase “confined to one small region,” the term does nat logically complete the sentence. 1 second half Explanation for Incorrect Answer C: Choice (C) is incorrect. In this context, “fading” describes a loss of strength or vitality ‘The sentence indicates that the outbreak of disease could not be labeled “pandemic,” or occurring over a wide geographic area and affecting an exceptionally high propartion of the population, The word “despite” in the second half of the sentence suggests that there will be a contrast between the missing word and the phrase “it remained confined to one small region." It makes sense to suggest that an outbreak of disease that Quickly fades, or loses strength, might be “confined to one small region”—thus, there is ro contrast. The term "fading" does not logicaly complete the sentence, Explanation fer Incorrect Answer D : From SAT Online Co urse i CLUS ABI ‘SAT 2006 May Answers and Explanations Choice (D) is incorrect. A symptom is something that indicates the presence of a disease or disorder, The sentence indicates that the outbreak of disease coul not be labeled "pandemic," or occurring over a wide geographic area and affecting an ‘exceptionally high proportion of the population, The word "despite" in the second half the sentence suggests that there will be a contrast between the missing word and the phrase “it remained confined to one small region." Although there may have been rapid "symptoms" caused by the disease (an Infected person might become very ill right away, for example), It does not make sense to say that the symptoms of the outbreak itself were rapic. Further, there is no logical connection between the symptoms of the disease and the confinement ofthe disease to “one small region.” Explanation for Incorrect Answer E choice (E) Is incorrect, In th's context, an “improvement” 's something that makes something better or something that makes progress toward what is desirable. The the outbreak of disease could not be labeled “pandemic,” or ‘occurring aver a wide geographic area and affecting an exceptionally high proportion of sentence indicates that the population. The word “despite” in the second half of the sentence suggests that between the missing word and the phrase "it remained confined to one small region.” Keeping the disease limited to one small area could be considered {2n improvement; therefore, the term “improvements” would not demonstrate the at Is indicated by the term “despite.” there will bea contra ‘The author's theory about modern design had an enormous impact when first published, but as, as it was then, itis now clearly ~ (scholarly. .erucite (©) lackluster .. Impressive WS) influential. outdated (©) primitive . . antiquarian (© prestigious . . masterful ANSWERS AND EXPLANATIONS: Explanation for Correct Answer C: Choice (C) is correct. “Influential” means having the power to change minds. “Outdated” means no longer current. The sentence suggests thet the Impact of the author's theory has changes since it was first published. The first missing word will escribe the theory “as it was then," and the second missing word will describe it as it Is now, Therefore, the two missing words will have contrasting meanings and will demonstrate this change. The sentence states that the theory "had an enormous Impact when first published,” so it certainly makes sense to say the theory was once Influential, or had the power to change minds. Something that is “outdatee," ar ne long From SAT Online Co urse CLUS HA ‘SAT 2006 May Answers and Explanations cr current, would most likely nat have the same impact that it once had, so it also makes sense to say that the ance-influental theory Is now outdated. Explanation for Incorrect Answer A: Choice (A) is Incorrect. “Scholarly” means learned or academic. "Erudite” means emonstrating extensive knowledge. The sentence suggests that the impact of the author's theory has changes since it was first published. The first missing word will describe the theory “asit was then,” and the second missing word will describe it as it Is now. Therefore, the two missing words will have contrasting meanings and will emonstrate this change. A theory that "had an enormous impact when first published could certainly be described as academic, so the term "scholarly" makes sense in the first blank. However, describing the theory as “erudite,” or demonstrating extensive knowledge, does not indicate that perceptions of the theory have changed over time, because itis similar t describing the theory as learned or academic. Explanation for Incorrect Answer B Choice (B) is incorrect. “Lackluster” means dull or lacking in brilliance. Something that Is timpressive” excites attention or admiration. The sentence suggests that the impact of the author's theory has changed since it was first published. The first missing wore will deseribe the theory "as it was then," and the second missing word will describe It as its now. Therefore, the two missing words will have contrasting meanings and wil emonstrate this change. Although these two terms indicate that perceptions of the theory have changed—that It was once considered lackluster, or Gull, and It is now considered impressive—it is unlikely that a theory that was considered lackluster when It was frst published would nave "had an enormous impact.” Explanation for Incorrect Answer Dt Choice (D) is incorrect. In this context, “primitive” means characteristic of an early stage of development, Something that is “antiquarian” relates to the life or culture of ancient times. The sentence suggests that the Impact of the author's theory has changed since it was frst published. The frst missing word will describe the theory “as it was then,” and the second missing word will describe it as it is now. Therefore, the two missing words wil have contrasting meanings and will demonstrate this change. It Is not logical to suggest that a theory that was considered primitive would have *had an enormous impact when first published,” or would even be published at all. Further, because the terms "primitive" and “antiquarian” are somewhat similar in meaning, they o not clearly indicate that perceptions of the theory have changed over time. Explanation for Incorrect Answer E: Choice (E) Is incorrect. Something that is “prestigious” is honored or held in high esteem. Something that is “masterful” demonstrates skill or full command of an area of FromSAT Online Course CUUS WABI

S-ar putea să vă placă și